You are on page 1of 88

For Student

BT is an 85-year-old man who has been admitted to the hospital for repair of a hip fracture he
sustained while playing golf. He has a history of hypertension and hypercholesterolemia, both of
which are treated. On review of systems, he admits to “memory problems” and difficulty sleeping
at night. The day after admission he undergoes total hip replacement. Blood loss is estimated at
300cc with two units of blood transfused during the surgery. He seems to recover well the next
day after surgery, and is talking with his family about “getting back on the course.”
Four days post-op, his wife voices concern that he is “not himself.” He seems to ignore her at times
during conversations and is not eating much. She says he is often confused, saying that he asks her
if she’s “taken the dog out” when they haven’t had a dog in years. That night he pulls his IV line
out and the nurse witnesses him trying to remove his Foley catheter.
Medications are: Aspirin 325mg po daily Atenolol 25mg po daily Simvastatin 20mg po qHS
Meperidine 50 mg q6h prn pain Diphenhydramine 25mg po qHS prn sleep
The patient’s vital signs are normal except for a temperature of 95.4 F. He seems relatively alert
but has difficulty answering questions. He is wearing wrist restraints but is surprised when you
point them out to him. The remainder of the exam is normal except for the presence of a foley
catheter and a clean, well-healing surgical site on the left hip

Tasks:
1. Identify medications the patient is taking that can contribute to this patient’s delirium, as
found in the APA Practice Guidelines.
2. ere there any pre-hospitalization risk factors before the patient’s admission?
3. Identify post-hospitalization risk factors that occurred during surgery, but before the
manifestation of delirium on post-op day 4.
Student’s name
For examiner
BT is an 85-year-old man who has been admitted to the hospital for repair of a hip fracture he
sustained while playing golf. He has a history of hypertension and hypercholesterolemia, both of
which are treated. On review of systems, he admits to “memory problems” and difficulty sleeping
at night. The day after admission he undergoes total hip replacement. Blood loss is estimated at
300cc with two units of blood transfused during the surgery. He seems to recover well the next
day after surgery, and is talking with his family about “getting back on the course.”
Four days post-op, his wife voices concern that he is “not himself.” He seems to ignore her at times
during conversations and is not eating much. She says he is often confused, saying that he asks her
if she’s “taken the dog out” when they haven’t had a dog in years. That night he pulls his IV line
out and the nurse witnesses him trying to remove his Foley catheter.
Medications are: Aspirin 325mg po daily Atenolol 25mg po daily Simvastatin 20mg po qHS
Meperidine 50 mg q6h prn pain Diphenhydramine 25mg po qHS prn sleep
The patient’s vital signs are normal except for a temperature of 95.4 F. He seems relatively alert
but has difficulty answering questions. He is wearing wrist restraints but is surprised when you
point them out to him. The remainder of the exam is normal except for the presence of a foley
catheter and a clean, well-healing surgical site on the left hip.

Task 1 (3.5) - Diphenhydramine is a high-risk agent likely to cause delirium from direct
anthistamine and indirect anticholinergic effects. Propoxyphene is a CNS active agent that may
also precipitate delirium, especially in conjunction with other centrally-active drugs. Atenolol has
poor blood-brain barrier penetrance and is unlikely to cause delirium as other beta blockers, such
as propranolol, can. Aspirin and simvastatin are not associated with delirium.
Task 2 (3.0) - The patient’s admitted memory disturbance and insomnia are potential markers of
previously existing cognitive impairment that may indicate early dementia, a strong risk factor for
delirium.
Task 3 (3.5) - Events that occurred in this patient’s first four days of hospilization that increased
his risk of delirium included:
a. Anemia due to blood loss
b. Anesthesia during surgery (there is no difference between local or general anesthesia)
c. Medications
d. Application of foley catheter
e. Application of restraints.
Final score:
For Student

An 84-year-old woman brought to hospital after being found wandering in the street at night by
the neighbours.
The patient is very disoriented and confused. She is alert but restless and agitated. She is convinced
that she is late for work. A telephone call to her niece confirms that the patient has been somewhat
confused and forgetful for the past 9 months or so. Further enquiry reveals that the patient needs
help with her household and financial affairs. She has never wandered before.
Examination discloses AF (heart rate 126/min), cardiomegaly, mitral incompetence and peripheral
oedema. Her feet are in a state of neglect and there is an infected bunion. Abbreviated mental test
is 4/10. There are no focal neurological signs. The remainder of the examination is unremarkable.

Tasks:
1. Give the most likely causes of the patient’s confusional state.
2. What initial investigations should you perform?
3. What treatment would you consider appropriate?
The patient’s niece is worried about her aunt returning home alone at the conclusion of the hospital
admission. How would you address this concern?

Student’s name
For examiner

An 84-year-old woman brought to hospital after being found wandering in the street at night by
the neighbours.
The patient is very disoriented and confused. She is alert but restless and agitated. She is convinced
that she is late for work. A telephone call to her niece confirms that the patient has been somewhat
confused and forgetful for the past 9 months or so. Further enquiry reveals that the patient needs
help with her household and financial affairs. She has never wandered before.
Examination discloses AF (heart rate 126/min), cardiomegaly, mitral incompetence and peripheral
oedema. Her feet are in a state of neglect and there is an infected bunion. Abbreviated mental test
is 4/10. There are no focal neurological signs. The remainder of the examination is unremarkable.
Task 1 (3.5) - The history is very suggestive of an acute-on-chronic confusional state, the former
precipitated by acute illness(es). Underlying cognitive impairment as a result of early Alzheimer’s
diseases is quite likely in this setting but must not be assumed. A search for acute illnesses such as
infection (UTI, infected bunion, respiratory), side effects of medication, heart failure and
metabolic disturbance (e.g. diabetes, uraemia) is essential. The patient was found wandering at
night, so hypothermia must be considered. In addition to an acute illness, causes of chronic
confusion must be considered, such as dementia (Alzheimer’s or vascular brain disease), thyroid
dysfunction, vitamin B12 deficiency and hypercalcaemia. If there is a poorly maintained gas
appliance at home, chronic carbon monoxide (CO) poisoning may need to be excluded.
Task 2 (3.0) - FBC – anaemia, infection
urinalysis and MSU – infection, diabetes
blood cultures and swab from infected bunion
plasma electrolytes, glucose and renal function – metabolic disturbance and uraemia
calcium – confusion caused by hypercalcaemia
thyroid function tests – often difficult to interpret in acute illness, but in this case important to
exclude thyroid dysfunction, particularly hyperthyroidism as a result of AF and heart failure
chest radiograph – infection, heart failure
ECG – to confirm AF
arterial oxygen saturation (and spectrophotometry for carboxyhaemoglobin if CO poisoning
suspected).
Once the acute illnesses have settled, and if chronic confusion persists, CT of the brain should be
considered to exclude potentially treatable causes of chronic confusion such as hydrocephalus,
meningioma and chronic subdural haematoma. Vitamin B12 and folate levels are needed if there
is macrocytic anaemia.
Task 3 (3.5) - This is a difficult issue and there is insufficient information provided to formulate
a full answer to this question. Once acute confusion has settled, the patient’s cognitive and
functional status should be reassessed by the multidisciplinary team (in particular, the occupational
therapist and social worker). An old-age psychiatry consultation may be required, particularly if a
new or former diagnosis of dementia is confirmed, so that an assessment of the patient’s mental
capacity can be made. If the patient insists on returning home alone, a formal judgement as per the
Mental Capacity Act 2005 concerning her capacity to assess the risk involved will be important.
If the patient is judged to have capacity regarding discharge destination, the risks associated with
returning home should be minimized by her family and the multidisciplinary team, including social
services if required. Assessment of the patient’s home environment will also be important before
a decision about discharge is finalized. If the patient lacks capacity to make decisions about her
discharge destination, a formal ‘best interests meeting’ as per the Mental Capacity Act will need
to be convened. Discharge home may still be the preferred option after full assessment.
Acetylcholinesterase inhibitor therapy (e.g. donepezil) to enhance cognition is contraindicated in
this patient because of significant cardiac disease.

Final score:

For Student

An 84-year-old woman has been brought to A&E, having been found wandering at night. The
patient is alert but confused (Mini-Mental State Examination [MMSE] 12/30 (normal 27+/30)).
There are no focal neurological signs, but she is cold (core temperature 35.5 ∞C). She is unable to
give a coherent history. You telephone the patient’s daughter to gain more historical information.
Give five key aspects of the history that you need to establish in discussion with the daughter.

Tasks:
Give five key aspects of the history that you need to
establish in discussion with the daughter.
Student’s name
For examiner
An 84-year-old woman has been brought to A&E, having been found wandering at night. The
patient is alert but confused (Mini-Mental State Examination [MMSE] 12/30 (normal 27+/30)).
There are no focal neurological signs, but she is cold (core temperature 35.5 ∞C). She is unable to
give a coherent history. You telephone the patient’s daughter to gain more historical information.
Give five key aspects of the history that you need to establish in discussion with the daughter.
In discussion with the patient’s daughter, you need to establish the following key aspects of the
history.
Task 1 (2.0) Duration of confusion: is this an acute, acute-on-chronic or chronic confusional state?
Acute and acute-on-chronic confusion suggest the presence of delirium and warrant a diligent
search for intercurrent general medical conditions that may have precipitated the current delirious
state (e.g. infection, CNS event such as head injury, metabolic disturbance, hypothyroidism). The
presence of chronic confusion (duration > 6 months) is suggestive of a dementing illness such as
Alzheimer’s disease.
Task 2 (2.0) Systems enquiry: the patient cannot give a coherent account of events, but while lucid
she may have previously reported important symptoms to a close relative or carer. Enquiry along
these lines may reveal, for example, a description of a recent head injury (subdural haematoma),
urinary symptoms Acute confusional states 167 (infection), thirst and urinary frequency (diabetes
mellitus), or cold intolerance (hypothyroidism). Any predisposing factors for hypothermia should
be sought.
Task 3 (2.0) Previous medical history: this may give important information relevant to the
patient’s current presentation – e.g. a history of liver disease, cerebrovascular disease, chronic
renal impairment or cardiopulmonary disease may cause or predispose to an acute confusional
state. You need to find out if there is a previous formal diagnosis of dementia or other psychiatric
history.
Task 4 (2.0) Drug history: medications commonly cause confusion in older patients. Offending
agents include anticholinergics (e.g. trihexyphenidyl for Parkinson’s disease), tricyclic
antidepressants (e.g. amitriptyline), opiates, hypoglycaemic agents (e.g. glibenclamide) and
dopaminergic drugs (e.g. L-dopa). Alcohol should also be considered. Polypharmacy (more than
five medications) can be the cause of a confusional state when many drugs, each with weak
anticholinergic properties, combine to produce appreciable central anticholinergic side effects (e.g.
digoxin, nitrates, warfarin, furosemide, ranitidine, cyclizine).
Task 5 (2.0) Social history: this information is essential to gain a proper understanding of the
context of the patient’s presenting illness, potential for rehabilitation and, in due course, discharge
planning.
Final score:
For Student
History
An 86-year-old man has been in a residential home for 3 years since his wife died. He was unable
to look after himself at home because of some osteoarthritis in the hips limiting his mobility. Apart
from his reduced mobility, which has restricted him to a few steps on a frame, and a rather irritable
temper when he doesn’t get his own way, he has had no problems in residential care. However, he
has become much more difficult over the last 36 h. He has accused the staff of assaulting him and
stealing his money. He has been trying to get out of his bed and his chair, and this has resulted in
a number of falls. On some occasions his speech has been difficult to understand. He has become
incontinent of urine over the last 24 h. Prior to this he had only been incontinent on one or two
occasions in the last 6 months. The duty doctor is called to see him and finds that he is rather
sleepy. When roused he seems frightened and verbally aggressive. He thinks that there is a
conspiracy in the ward and that the staff are having secret meetings and planning to harm him. He
is disorientated in place and time although reluctant to try to answer these questions. He is a non-
smoker and drinks 1–2 units a month. On a routine blood test 8 years ago he was diagnosed with
hypothyroidism and thyroxine 100 mg daily is the only medication he is taking. The staff say that
he has taken this regularly up to the last 36 h and his records show that his thyroid function was
normal when it was checked 6 months earlier. The staff say that he is now too difficult to manage
in the residential home. They feel that he has dementia and that the home is not an appropriate
place for such patients.
Examination
There is nothing abnormal to find apart from blood pressure of 178/102 mmHg and limitation of
hip movement with pain and a little discomfort in the right loin
Thyroxine 125 nmol/L (70–140 nmol/L)
Thyroid-stimulating hormone 1.6 mU/L (0.3–6.0 mU/L)
Blood glucose 6.2 mmol/L (4.0–6.0 mmol/L)
Urine dipstick: – sugar, ' protein, '' blood

Tasks:
What should be done?

Student’s name
For examiner
History
An 86-year-old man has been in a residential home for 3 years since his wife died. He was unable
to look after himself at home because of some osteoarthritis in the hips limiting his mobility. Apart
from his reduced mobility, which has restricted him to a few steps on a frame, and a rather irritable
temper when he doesn’t get his own way, he has had no problems in residential care. However, he
has become much more difficult over the last 36 h. He has accused the staff of assaulting him and
stealing his money. He has been trying to get out of his bed and his chair, and this has resulted in
a number of falls. On some occasions his speech has been difficult to understand. He has become
incontinent of urine over the last 24 h. Prior to this he had only been incontinent on one or two
occasions in the last 6 months. The duty doctor is called to see him and finds that he is rather
sleepy. When roused he seems frightened and verbally aggressive. He thinks that there is a
conspiracy in the ward and that the staff are having secret meetings and planning to harm him. He
is disorientated in place and time although reluctant to try to answer these questions. He is a non-
smoker and drinks 1–2 units a month. On a routine blood test 8 years ago he was diagnosed with
hypothyroidism and thyroxine 100 mg daily is the only medication he is taking. The staff say that
he has taken this regularly up to the last 36 h and his records show that his thyroid function was
normal when it was checked 6 months earlier. The staff say that he is now too difficult to manage
in the residential home. They feel that he has dementia and that the home is not an appropriate
place for such patients.
Examination
There is nothing abnormal to find apart from blood pressure of 178/102 mmHg and limitation of
hip movement with pain and a little discomfort in the right loin
Thyroxine 125 nmol/L (70–140 nmol/L)
Thyroid-stimulating hormone 1.6 mU/L (0.3–6.0 mU/L)
Blood glucose 6.2 mmol/L (4.0–6.0 mmol/L)
Urine dipstick: – sugar, ' protein, '' blood
Task 1 (10.0) This is not the picture of dementia. The acute onset with clouding of consciousness,
hallucinations, delusions, restlessness and disorientation suggest an acute confusional state,
delirium. There are many causes of this state in the elderly. It can be provoked by drugs, infections,
metabolic or endocrine disorders, or other underlying conditions in the heart, lungs, brain or
abdomen.
There is no record of any drugs except thyroxine, although this should be rechecked to rule out
any analgesics or other agents that he might have had access to or that might not be regarded as
important.
The thyroid abnormality is not likely to be relevant. The lack of replacement for 2 days will not
have a significant effect and the normal results 6 months earlier make this an unlikely cause of his
current problem. The sugar is normal. Other metabolic causes such as renal failure, anaemia,
hyponatraemia and hypercalcaemia need to be excluded.
The falls raise the possibility of trauma, and a subdural haematoma could present in this way.
However, it seems that the falls were a secondary phenomenon. The most likely cause is that he
has a urinary tract infection. There is blood and protein in the urine, he has become incontinent
and he has some tenderness in the loin which could fit with pyelonephritis. We are not told whether
he had a fever, and the white cell count should be measured.
If this does seem the likely diagnosis it would be best to treat him where he is, if this is safe and
possible. He is likely to be more confused by a move to a new environment in hospital. There is
every likelihood that he will return to his previous state if the urinary tract infection is confirmed
and treated appropriately, although this may take longer than the response in temperature and white
cell count. Treatment should be started on the presumption of a urinary tract infection, while the
diagnosis is confirmed by microscopy and culture of the urine. The most likely organism is
Escherichia coli, and an antibiotic such as trimethoprim would be appropriate, although resistance
is possible and advice of the local microbiologist may be helpful. From the confusion point of view
he should be treated calmly, consistently and without confrontation. If medication is necessary,
small doses of a neuroleptic such as haloperidol or olanzapine would be appropriate.

Final score:

For Student
Mr. R is a 75-year-old man who comes to see you in clinic accompanied by his wife because she
is concerned that his memory is getting worse. She states that, for the last few months, he has been
getting lost driving 20 miles from his home to his local VA hospital where he volunteers. He has
done this job twice weekly for 25 years.
Mr. R’s past medical history is notable for chronic leg pain resulting from a war injury. He also
has a history of ischemic bowel, which has been asymptomatic since a hemicolectomy 3 years ago,
and gout. His medications are 1. Paroxetine, 20 mg daily 2. Methadone, 20 mg 3 times a day 3.
Meloxicam, 7.5 mg daily, orally 4. Acetaminophen with codeine (300/60), 2 tablets 3 times a day
5. Allopurinol 300 mg daily, orally
Mr. R’s exam thus far reveals some difficulty with recalling recent events. Given his age, his
baseline risk of dementia is at least 10%. The first step in his work-up would be to screen for
dementia with the MMSE or MIS. If this is positive, an effort should be made to see if he fulfills
the NINCDS-ADRDA criteria for probable AD.
Further history revealed that the patient’s wife had taken over bookkeeping because a few bills
had gone unpaid during the last 3 months. The patient was given the MMSE and scored a 20 out
of 30. He was not able to give the day of the month, could only register 2 of 3 items and recalled
0 of 3. He only got 1 of the serial 7s and could not draw pentagons. Consideration of the NINCDS-
ADRDA criteria showed him to have dementia with deficits in 2 or more areas of cognition
(orientation, visuospatial and executive functioning, attention and working memory, and memory).
At the time of the visit, it was not clear whether his cognitive functioning was worsening and there
were no disturbances in consciousness. The plan was made for initial laboratory work to be done
and for a 3-month follow-up visit. Given that he was taking multiple psychoactive medications,
his regimen was scaled back to the minimum doses necessary to control his pain.

Tasks:

1. At this point, what is the leading hypothesis, what are the active alternatives, and is there a
must not miss diagnosis? Given this differential diagnosis, what tests should be ordered?
2. Have you crossed a diagnostic threshold for the leading hypothesis, AD? Have you ruled
out the active alternatives? Do other tests need to be done to exclude the alternative
diagnoses?

Student’s name
For examiner
Mr. R is a 75-year-old man who comes to see you in clinic accompanied by his wife because she
is concerned that his memory is getting worse. She states that, for the last few months, he has been
getting lost driving 20 miles from his home to his local VA hospital where he volunteers. He has
done this job twice weekly for 25 years.
Mr. R’s past medical history is notable for chronic leg pain resulting from a war injury. He also
has a history of ischemic bowel, which has been asymptomatic since a hemicolectomy 3 years ago,
and gout. His medications are 1. Paroxetine, 20 mg daily 2. Methadone, 20 mg 3 times a day 3.
Meloxicam, 7.5 mg daily, orally 4. Acetaminophen with codeine (300/60), 2 tablets 3 times a day
5. Allopurinol 300 mg daily, orally
Mr. R’s exam thus far reveals some difficulty with recalling recent events. Given his age, his
baseline risk of dementia is at least 10%. The first step in his work-up would be to screen for
dementia with the MMSE or MIS. If this is positive, an effort should be made to see if he fulfills
the NINCDS-ADRDA criteria for probable AD.
Further history revealed that the patient’s wife had taken over bookkeeping because a few bills
had gone unpaid during the last 3 months. The patient was given the MMSE and scored a 20 out
of 30. He was not able to give the day of the month, could only register 2 of 3 items and recalled
0 of 3. He only got 1 of the serial 7s and could not draw pentagons. Consideration of the NINCDS-
ADRDA criteria showed him to have dementia with deficits in 2 or more areas of cognition
(orientation, visuospatial and executive functioning, attention and working memory, and memory).
At the time of the visit, it was not clear whether his cognitive functioning was worsening and there
were no disturbances in consciousness. The plan was made for initial laboratory work to be done
and for a 3-month follow-up visit. Given that he was taking multiple psychoactive medications,
his regimen was scaled back to the minimum doses necessary to control his pain.
Task 1 (5.0) Mr. R has had a decline in cognitive status. He is unable to do a higher-level task that
he used to do. Given that this patient is exhibiting cognitive decline, dementia—most commonly
Alzheimer disease (AD)—has to be included in the differential diagnosis. The subacute onset of
this patient’s symptoms, with loss of recall, makes AD likely. Another common cause of dementia
in older persons is vascular dementia (VaD). It will be important to determine whether this patient
has risk factors for cerebrovascular disease. In an older person, clinicians have to consider the
normal cognitive decline that comes with aging, but normal cognitive aging never causes
functional compromise. An alternative diagnosis is mild cognitive impairment (MCI), a syndrome
of memory loss more severe than the memory loss that occurs with normal aging. MCI, however,
also does not cause functional decline. Delirium and depression should always be considered in an
older patient with cognitive decline because they are highly treatable. A patient who is unable to
successfully live independently because of cognitive issues always has an abnormality.
Task 2 (5.0) Alternative Diagnosis: Multi-infarct Dementia (Vascular Dementia, VaD). A patient
with VaD may have dementia that has an abrupt onset or is slowly worsening. The patient usually
has risk factors for vascular disease or has previously diagnosed vascular disease. The patient often
has difficulty walking or a focal neurologic exam.
Evidence-Based Diagnosis the clinical diagnosis of VaD include: 1. The development of cognitive
deficits including memory deficits 2. A resulting significant impairment in social or occupational
functioning 3. Focal neurologic signs, symptoms, or diagnostic studies indicative of
cerebrovascular disease judged to be etiologically related to the cognitive change.
DLB is typically seen in a patient with Parkinson disease who has dementia. The predominant
symptoms of the dementia are a fluctuating course and the presence of hallucinations. In patients
without a previous diagnosis of Parkinson disease, motor symptoms similar to those seen in
Parkinson disease are often present.

Final score:

For Student
Mrs DM Scenario This is an 83 year old lady diagnosed with multi infarct dementia in 2008. Lives
with husband and the couple are supported by family members. Husband reports that his wife is
getting worse and is frequently waking up at night and getting up. She misplaces items and then
accuses family member of taking them. Over the last few months she has started to see young
people in the trees in the garden. She is worried in case they fall out. More recently she is hearing
the sound of a child crying which she finds quite distressing and has also seen the child down the
road. Her relatives were aggravating her by insisting to her that her experiences were unreal.
Current Medications: simvastatin 40mg od, aspirin 75 mg od medical history; nil of note

Tasks:
1.Medical review to rule out acute on chronic confusion:
2. Identify ways by which her husband could be supported in her care.
Student’s name
For examiner
Mrs DM Scenario This is an 83 year old lady diagnosed with multi infarct dementia in 2008. Lives
with husband and the couple are supported by family members. Husband reports that his wife is
getting worse and is frequently waking up at night and getting up. She misplaces items and then
accuses family member of taking them. Over the last few months she has started to see young
people in the trees in the garden. She is worried in case they fall out. More recently she is hearing
the sound of a child crying which she finds quite distressing and has also seen the child down the
road. Her relatives were aggravating her by insisting to her that her experiences were unreal.
Current Medications: simvastatin 40mg od, aspirin 75 mg od medical history; nil of note

Task 1 (5.0) Medical review Bloods and MSU - normal Prescribed Risperdal 0.25mg nocte
licensed for agitation in dementia. Only prescribed after discussing risks of taking this medication
with her relatives and explaining possible side effects.
Task 2 (5.0) Support husband with advice not to challenge her about these hallucinations as they
are real to her. Continued use will be reviewed within 12 weeks. Social support. Relatives were
provided with psycho-education on dementia to inform them on what to expect and provide them
with coping strategies such as gentle reality orientation.

Final score:

For Student
JR is a 70-year-old man who is brought into your outpatient clinic by his 45-year-old daughter
(BG) because of a perceived inability to care for him at home. She states that he is generally
healthy, but over the course of the last 2 to 3 years she has noticed changes in his memory and
behavior. She is mostly concerned about his wandering and need for constant supervision.
On history, she volunteers that she first noticed “something was off” when he started getting lost
in the supermarket on grocery trips by himself. He used to be an excellent cook, but now can barely
make himself a sandwich. JR also sometimes doesn’t recognize his grandchildren--- last week he
yelled at his 10 year old grandson to “get off my property”.
JR’s past medical history includes hypertension, coronary artery disease with a myocardial
infarction 8 years ago. His medications include aspirin 81 mg daily, atenolol 25 mg daily, and
hydrochlorothiazide 25 mg daily.
JR needs some help with bathing. As for other basic ADLs1, he can do his own transfers, feed,
and toilet himself; with guidance, he can do his own grooming and dressing. However, he is almost
completely dependent in instrumental activities of daily living (IADLs).
As for his physical exam, vital signs are within normal limits. The rest of the physical exam is
normal, including an evaluation of gait and balance. When you perform a MiniCog2 evaluation,
he is able to only remember 1 out of 3 objects you asked him to retain. When asked to draw the
face of a clock, he jumbles all of the numbers together on one side of the clock face and is unable
to figure out how to draw the hands BG appears anxious throughout the whole interview and keeps
repeating statements like “I don’t know what else to do with him”. She is an only child and has no
one else to help her with her father.

Tasks:
1. Develop a management plan that incorporates a global approach, including medication and
social interventions.
2. Use overview for discussion on pharmacologic treatment, and you can also link to the
Family Caregiver

Student’s name
For examiner
JR is a 70-year-old man who is brought into your outpatient clinic by his 45-year-old daughter
(BG) because of a perceived inability to care for him at home. She states that he is generally
healthy, but over the course of the last 2 to 3 years she has noticed changes in his memory and
behavior. She is mostly concerned about his wandering and need for constant supervision.
On history, she volunteers that she first noticed “something was off” when he started getting lost
in the supermarket on grocery trips by himself. He used to be an excellent cook, but now can barely
make himself a sandwich. JR also sometimes doesn’t recognize his grandchildren--- last week he
yelled at his 10 year old grandson to “get off my property”.
JR’s past medical history includes hypertension, coronary artery disease with a myocardial
infarction 8 years ago. His medications include aspirin 81 mg daily, atenolol 25 mg daily, and
hydrochlorothiazide 25 mg daily.
JR needs some help with bathing. As for other basic ADLs1, he can do his own transfers, feed,
and toilet himself; with guidance, he can do his own grooming and dressing. However, he is almost
completely dependent in instrumental activities of daily living (IADLs).
As for his physical exam, vital signs are within normal limits. The rest of the physical exam is
normal, including an evaluation of gait and balance. When you perform a MiniCog2 evaluation,
he is able to only remember 1 out of 3 objects you asked him to retain. When asked to draw the
face of a clock, he jumbles all of the numbers together on one side of the clock face and is unable
to figure out how to draw the hands BG appears anxious throughout the whole interview and keeps
repeating statements like “I don’t know what else to do with him”. She is an only child and has no
one else to help her with her father.

Task 1 (5.0) It’s important to approach the issue from a biopsychosocial perspective. Because of
his need for constant supervision, as well as some ADL needs, he could be classified as having a
moderate dementia. He could be a candidate for cholinesterase inhibitors and memantine.
Task 2 (5.0) He also needs caregiver support and needs community resources, such as adult day
care and home care. You should also encourage her to enlist other family support.

Final score:

For Student
Mr MB Scenario This is an 81 year old man with severe AD (Alzheimer Disease) dementia. He
has been living alone for the last 6 years since his wife died. He has a carer who attends to his
welfare between the hours of 8:30am and 5:30 pm 6 days a week. He is generally amiable with his
main carer but has been known to throw water over her on occasions out of the blue. On the one
day that this carer is not around a care agency sends in a carer to observe the same routine as his
main carer. He has been known to be verbally and physically aggressive to the “one day a week
carers” and has been known to walk around in the neighbourhood on the days when they attend to
his care. In spite of his severe expressive dysphasia, severe decline in his power of recall and total
dependence on his carers for his basic hygiene and dressing, he remains able to find his way around
in the neighbourhood.
He was admitted to hospital in the interest of his health and safety and safety of others. Recall
patient has no mental capacity to cater for himself.

Tasks:
1. Revise his care package and medical treatment. Use his hospital admission as an
opportunity to put together a package to accommodate his needs 24 hour supported
accommodation for supervision and support he requires. assess his response to medications
aimed at reducing his verbal and physical aggression.
2. Any changes and follow up?

Final score:

Student’s name
For examiner
Mr MB Scenario This is an 81 year old man with severe AD (Alzheimer Disease) dementia. He
has been living alone for the last 6 years since his wife died. He has a carer who attends to his
welfare between the hours of 8:30am and 5:30 pm 6 days a week. He is generally amiable with his
main carer but has been known to throw water over her on occasions out of the blue. On the one
day that this carer is not around a care agency sends in a carer to observe the same routine as his
main carer. He has been known to be verbally and physically aggressive to the “one day a week
carers” and has been known to walk around in the neighbourhood on the days when they attend to
his care. In spite of his severe expressive dysphasia, severe decline in his power of recall and total
dependence on his carers for his basic hygiene and dressing, he remains able to find his way around
in the neighbourhood.
He was admitted to hospital in the interest of his health and safety and safety of others. Recall
patient has no mental capacity to cater for himself.

Task 1 (5.0) Withdrawn of medication Olanzapine. He was initially placed on Olanzapine by his
GP to help address his behaviour to his “one day carers” but he became excessively sedated on
this medication. This medication was withdrawn and changed to Sodium Valproate which seemed
to calm him down and make him more amenable to care until he pulled a knife on one on the “one
day carers”.
Task 2 (5.0). Review his physical health. Rule out “acute on chronic” confusional state.

For Student
Mrs A: Scenario This is an elderly frail female patient with dementia who was known to local
community mental health trust and a private GP. She lived alone, was a life long smoker and was
admitted to hospital with fractured neck of femur. She was extremely anxious on the ward and
refused to engage with rehabilitation .
Drugs on admission: – quetiapine, mirtazapine, thyroxine, and pregabalin.
Tasks:
1.What might have happened to her?
2. What else can be explored?

Student’s name
For examiner
Mrs A: Scenario This is an elderly frail female patient with dementia who was known to local
community mental health trust and a private GP. She lived alone, was a life long smoker and was
admitted to hospital with fractured neck of femur. She was extremely anxious on the ward and
refused to engage with rehabilitation .
Drugs on admission: – quetiapine, mirtazapine, thyroxine, and pregabalin.
Task 1 (5.0)
Delirium identified, anxiety can be reduced by ensuring good sensory awareness by ensuring
hearing aid and her spectacles were available to her
Encouraging engagement in activities (e.g. reminiscence/familiar object around her – Encouraging
good sleep pattern (using milky drinks at bedtime, exercise during the day if possible ) rather than
using night sedation.
Advice was offered to family and ward about delirium and management.
Task 2 (5.0) What else can be explored? – Questionable prescriptions –
Meds rec revealed that pregabalin had been stopped several months previously.
Nicotine withdrawal – Patches were provided –
Adverse drug effects that could be contributing to her conditions. She had hyponatraemia
aggravated by antidepressants,mental state improved when corrected with delirium resolving. –
Community team was recommended to review quetiapine and mirtazapine after discharge.

Final score:

For Student
Mrs A: Scenario This is an elderly frail female patient with dementia who was known to local
community mental health trust and a private GP. She lived alone, was a life long smoker and was
admitted to hospital with fractured neck of femur. She was extremely anxious on the ward and
refused to engage with rehabilitation .
Drugs on admission: – quetiapine, mirtazapine, thyroxine, and pregabalin.

Tasks:
1.What might have happened to her?
2. What else can be explored?
Student’s name
For examiner
Mrs A: Scenario This is an elderly frail female patient with dementia who was known to local
community mental health trust and a private GP. She lived alone, was a life long smoker and was
admitted to hospital with fractured neck of femur. She was extremely anxious on the ward and
refused to engage with rehabilitation .
Drugs on admission: – quetiapine, mirtazapine, thyroxine, and pregabalin.

Task 1 (5.0)
Delirium identified, anxiety can be reduced by ensuring good sensory awareness by ensuring
hearing aid and her spectacles were available to her
Encouraging engagement in activities (e.g. reminiscence/familiar object around her – Encouraging
good sleep pattern (using milky drinks at bedtime, exercise during the day if possible ) rather than
using night sedation.
Advice was offered to family and ward about delirium and management.
Task 2 (5.0) What else can be explored? – Questionable prescriptions –
Meds rec revealed that pregabalin had been stopped several months previously.
Nicotine withdrawal – Patches were provided –
Adverse drug effects that could be contributing to her conditions. She had hyponatraemia
aggravated by antidepressants,mental state improved when corrected with delirium resolving. –
Community team was recommended to review quetiapine and mirtazapine after discharge.

Final score:

For Student
ST is an 87-year-old woman who is brought by her son to your office to establish primary care.
The son recently moved his mother here from Florida because of increasing concerns about her
health and safety of her living situation. The medical problems listed on the outside medical
records that come with the patient include: hypertension, sleep problems (both initiating and
maintaining sleep), degenerative joint disease, peripheral edema, mild dementia, intermittent
daytime urge type urinary incontinence, and gait problems, but without history of falls. New
concerns that the son has about his mother are (1) possible depression - her husband died 10 months
ago and her social interactions have decreased, her hygiene has worsened, and she seems more
apathetic; and (2) her inability to take her numerous medications correctly.
Her current medication list is:
Captopril 12.5 mg t.i.d.
Nifedipine 10 mg t.i.d.
Furosemide 20 mg b.i.d.
KCl 10 meq b.i.d.
Digoxin 0.125 mg daily
Cimetidine 100 mg qhs
Indomethacin 25 mg b.i.d.
Diphenhydramine 50 mg qhs
Gabapentin 300mg b.i.d
Review of systems is negative for diabetes mellitus, congestive heart failure (outside
echocardiogram report shows LVEF 55% with mild diastolic dysfunction), cardiac arrhythmias
(except for one episode of atrial fibrillation 14 years ago during her hospitalization for
cholecystectomy), GERD, peptic ulcer disease, or peripheral neuropathy.
Physical examination shows a blood pressure of 135/84 without orthostatic changes, regular pulse
of 76, normal cardiopulmonary findings, findings of chronic DJD of knees 36 and hands, trace
bilateral ankle edema, somewhat depressed affect, Mini-mental Status Exam (MMSE) of 25 (out
of possible 30), and Geriatric Depression Scale score of 9 (>5 consistent with depression).

Tasks:
1. What current medications of the patient’s are considered potentially inappropriate to use
in the elderly?
2. Which of the patient’s medications do not have a specific indication for their use?
3. Which current medications might be exacerbating or precipitating some of the patient’s
problems? What patient problems are not being treated by her current medical regimen?
4. Identify at least 5 medication changes you could make that could improve the medical
treatment and medication compliance of the patient.
Student’s name
For examiner
ST is an 87-year-old woman who is brought by her son to your office to establish primary care.
The son recently moved his mother here from Florida because of increasing concerns about her
health and safety of her living situation. The medical problems listed on the outside medical
records that come with the patient include: hypertension, sleep problems (both initiating and
maintaining sleep), degenerative joint disease, peripheral edema, mild dementia, intermittent
daytime urge type urinary incontinence, and gait problems, but without history of falls. New
concerns that the son has about his mother are (1) possible depression - her husband died 10 months
ago and her social interactions have decreased, her hygiene has worsened, and she seems more
apathetic; and (2) her inability to take her numerous medications correctly.
Her current medication list is:
Captopril 12.5 mg t.i.d.
Nifedipine 10 mg t.i.d.
Furosemide 20 mg b.i.d.
KCl 10 meq b.i.d.
Digoxin 0.125 mg daily
Cimetidine 100 mg qhs
Indomethacin 25 mg b.i.d.
Diphenhydramine 50 mg qhs
Gabapentin 300mg b.i.d
Review of systems is negative for diabetes mellitus, congestive heart failure (outside
echocardiogram report shows LVEF 55% with mild diastolic dysfunction), cardiac arrhythmias
(except for one episode of atrial fibrillation 14 years ago during her hospitalization for
cholecystectomy), GERD, peptic ulcer disease, or peripheral neuropathy.
Physical examination shows a blood pressure of 135/84 without orthostatic changes, regular pulse
of 76, normal cardiopulmonary findings, findings of chronic DJD of knees 36 and hands, trace
bilateral ankle edema, somewhat depressed affect, Mini-mental Status Exam (MMSE) of 25 (out
of possible 30), and Geriatric Depression Scale score of 9 (>5 consistent with depression).
Task 1 (3.0) Task 2 (2.0) According to the ACOVE indicators for Medication Use in the Elderly,
all medications need to have a specific indication for their use. Neither digoxin (which was given
for one peri-operative event of atrial fibrillation that occurred many years prior without further
episodes) nor cimetidine have a current indication for use, and discontinuation is recommended.
The indication for gabapentin is also unclear; though may be related to pain management, there is
no evidence for neuropathy, per se.
Task 3 (2.0) It is possible that the nifedipine and/or the gabapentin precipitated or exacerbated the
peripheral edema (leading to the need for furosemide, which in turn led to the need to add KCl).
The furosemide may be exacerbating the urinary incontinence. The patient has untreated
depression and unaddressed cognitive dysfunction, with difficulty correctly taking her complicated
medication regimen. The gabapentin, cimetidine, indomethicin, and diphenhydramine could all be
exacerbating her cognitive dysfunction.
Task 4 (3.0) Change the t.i.d dosed captopril to a once daily dose of another ACE-inhibitor (such
as lisinopril).
b. Patient has no specific reason to be on nifedipine other than hypertension and is treated with
another antihypertensive (ACE-inhibitor) at less than maximum effective dose. In addition, the
nifedipine may be exacerbating/precipitating the peripheral edema. Discontinue the nifedipine. c.
Taper the gabapentin and monitor for increased pain.
c. Once nifedipine is discontinued and gabapentin is discontinued (or at least minimized ), try
stopping furosemide (and KCl), as she may not need it.
d. Discontinue digoxin and cimetidine, as there are no current indications for their use.
e. Discontinue indomethicin and propoxyphene, as they may be exacerbating patient’s cognitive
status and the indomethicin may have increased renal toxicity in this elderly patient. May add
acetaminophen twice a day (with food) NSAID such as naprosyn, if needed for osteoarthritis pain.
f. Discontinue diphenhydramine and consider beginning an agent such as mirtazapine, which is an
antidepressant with sleep-inducing properties (i.e., one medication to treat two medical problems).

Final score:
For Student
An 84-year-old man is agitated and restless.
The patient is disturbing other residents in the residential care home. He has a history of cardiac
and cerebrovascular disease with vascular dementia. The on-call doctor who attends prescribes
haloperidol 2.5 mg stat and then 1 mg three times a day to follow.

Tasks:
1. What iatrogenic illness may result from the given intervention?
2. What risk factors for iatrogenic illness are present in this case?
3. Suggest some alternative management strategies.
4. How can these complications be minimized?
Student’s name
For examiner
The patient is disturbing other residents in the residential care home. He has a history of cardiac
and cerebrovascular disease with vascular dementia. The on-call doctor who attends prescribes
haloperidol 2.5 mg stat and then 1 mg three times a day to follow.
Task 1 (3.0) Short-term side effects may include hypotension, sedation and immobility, increasing
the risk of dehydration, poor nutrition and pressure sores. If continued for more than several days,
there is a risk of drug-induced parkinsonism and falls. If continued long term, there is an increased
risk of stroke, accelerated cognitive decline and continued parkinsonism. Mortality is increased
when neuroleptics are continued long term in such situations.
Task 2 (2.0) Age, cardiac disease and vascular brain disease increase the risk of adverse events.
Task 3 (2.0) If the agitation and restlessness are of recent onset, this is hyperactive delirium and
there are underlying causes, such as infection, pain, discomfort or metabolic disturbance. Such
precipitants should be carefully identified and treated as appropriate. Non-pharmacological means
to settle the patient’s agitation should be tried, including reorientation, the calming presence of a
family member or familiar carer, encouraging the patient to wear his glasses and hearing aid if
required, and hydration. Every opportunity to avoid pharmacological sedation should be
considered.
Task 4 (3.0) If sedation is required, then the initial dose of haloperidol should be kept low (1 mg
initially); if necessary, a further dose can be given 20–30 min later. Follow-on medication should
be given only if necessary, and in this situation should be low dose (0.5 mg twice a day). Treatment
should not be continued beyond 3 days and should never be continued indefinitely. If the patient
has pre-existing parkinsonism, a small dose of lorazepam 0.5–1 mg can be considered as an
alternative.

Final score:

For Student
An 84-year-old woman with dementia is admitted from a residential home because she has had
some falls. The patient is described as being frail. She is restless and agitated and has a respiratory
infection. Her skin is fragile, with several small lacerations and bruises caused by her recent falls.
The electronic prescribing process prompts the admitting medical team to assess for venous
thromboembolism (VTE) Iatrogenic illness in frail older patients 177 risk. She is considered at
risk because of her age, dehydration and infection. Should she be prescribed VTE prophylaxis with
subcutaneous enoxaparin?

Tasks:
1. What iatrogenic illness may result from the given intervention?
2. What risk factors for iatrogenic illness are present in this case?
3. Suggest some alternative management strategies.
4. How can these complications be minimized?

Student’s name
For examiner
An 84-year-old woman with dementia is admitted from a residential home because she has had
some falls. The patient is described as being frail. She is restless and agitated and has a respiratory
infection. Her skin is fragile, with several small lacerations and bruises caused by her recent falls.
The electronic prescribing process prompts the admitting medical team to assess for venous
thromboembolism (VTE) Iatrogenic illness in frail older patients 177 risk. She is considered at
risk because of her age, dehydration and infection. Should she be prescribed VTE prophylaxis with
subcutaneous enoxaparin?
Task 1 (3.0) Head injury and intracranial bleeding are a substantial risk because of the patient’s
frailty, confusion and risk of further falls. Low-dose enoxaparin will substantially increase this
risk. The distress caused by subcutaneous injections may exacerbate the patient’s delirium. There
may be cutaneous bruising at injection sites and elsewhere due to the patient’s restlessness and
agitation and resulting minor trauma.
Task 2 (2.0) Fall-related head injury carries a higher than normal risk of intracranial bleeding,
particularly subdural haemorrhage, because of age- and dementia-associated cerebral atrophy.
Task 3 (2.0) The patient could be assessed for anti-thromboembolic stockings if peripheral
circulation is satisfactory. Rehydration, treatment of respiratory infection and supervised
mobilization will reduce the risk of VTE.
Task 4 (3.0) The risk of VTE has to be balanced against the substantial risk (in this case) of harm
from increased bleeding. A supportive nursing environment and medical management aiming to
reverse treatable conditions and resolve delirium are important. If the patient is judged not to have
capacity to participate in treatment-related decisions, then the medical team should act in her best
interests. Timely involvement of family members or any legally appointed representative (lasting
power of attorney) is imperative.

Final score:

For Student
A frail 92-year-old woman has fallen at home. The ambulance service has conveyed her to the
local A&E department for further assessment. Further enquiry reveals the patient lives alone and
has mild dementia and previous osteoporosis-related fragility fractures (wrist and thoracic
vertebra). She has poor vision and hearing. She receives help from local family and the home-care
service. She has not sustained any bone injury. Physiological parameters are stable. She is taking
diuretics for ankle swelling. There are concerns about her safety at home. Should she be admitted
to hospital?

Tasks:
1. What iatrogenic illness may result from the given intervention?
2. What risk factors for iatrogenic illness are present in this case?
3. Suggest some alternative management strategies.
4. How can these complications be minimized?

Student’s name
For examiner

A frail 92-year-old woman has fallen at home. The ambulance service has conveyed her to the
local A&E department for further assessment. Further enquiry reveals the patient lives alone and
has mild dementia and previous osteoporosis-related fragility fractures (wrist and thoracic
vertebra). She has poor vision and hearing. She receives help from local family and the home-care
service. She has not sustained any bone injury. Physiological parameters are stable. She is taking
diuretics for ankle swelling. There are concerns about her safety at home. Should she be admitted
to hospital?
Task 1 (3.0) This a common scenario. The patient is frail and will be at substantial risk from
hospital admissionrelated complications such as delirium, further falls, health-care-associated
infection, deconditioning due to reduced activity and loss of functional skills. Local support
networks will be destabilized by her admission. These risks have to be balanced against the hazards
of returning to a potentially unsafe environment and occult illness.
Task 2 (2.0) The presence of frailty, cognitive impairment and sensory impairments substantially
increases the risk of delirium and falls within the hospital environment. Any subsequent ward
moves with unfamiliar routines and interventions will add to this risk.
Task 3 (2.0) Assessment by a rapid-response therapy and nursing team skilled in the assessment
of frail older people is essential in this A&E setting. The patient should have a supported discharge
home with reinstatement of her existing care plan, supplemented by short-term additional social
and therapy support as necessary. There should be further early review at home or attendance at a
community or day hospital-based falls clinic (depending on local availability of services) for
further comprehensive geriatric assessment. This will identify modifiable conditions and
implement treatment and support to enhance care at home and reduce risk of further falls.
Task 4 (3.0) Safe alternatives to hospital admission should always be considered in frail older
people. A working knowledge of local services and community teams is essential. Close liaison
and reassurance to the patient’s family and carers are also important. The medical role in A&E is
to quickly exclude lifethreatening or serious illness, and this will provide focus to the rapid-
response nursing/therapy team to permit early supported discharge. Many such teams are
supported by elderly-care physicians who are able to give further confidence with respect to
medical stability, immediate changes to medication and follow-up plans.

Final score:

For Student
An 82-year-old woman presents with recurrent falls.
The patient is housebound. Her most recent fall occurred while getting up to go to the toilet at
night. She has arthritis, heart failure and poor vision as a result of macular degeneration, and she
has had a previous hip replacement. She receives treatment with furosemide, ramipril, co-codamol,
amitriptyline and temazepam. She is very frail. The main findings on examination are: abbreviated
mental test 6/10, vision – large print only, kyphotic spine, brisk reflexes bilaterally, peripheral
oedema, clear lung fields and normal jugular venous pressure (JVP), heart rate 112/min, atrial
fibrillation (AF), blood pressure sitting 114/62 mmHg, mitral regurgitation.
Tasks:
1. What is the differential diagnosis?
2. What features in the history support the diagnosis?
3. What additional features in the history would you seek to support the potential diagnoses?
4. What other features would you look for on clinical examination?
5. What investigations would you perform?
6. What treatment options are available?

Student’s name
For examiner

Task 1 (2.0) - There are multiple causes, including: poor vision postural hypotension
polypharmacy neurological dysfunction – previous stroke, cervical myelopathy, vitamin B12
deficiency arthritis/osteomalacia arrhythmia – AF.
Task 2 (2.0) - Poor vision: the fall occurred at night when environmental hazards are more difficult
to see and avoid. Postural hypotension: the fall occurred as the patient was getting out of bed to
go to the toilet. She is on a lot of medications that can impair postural blood pressure control – a
diuretic, an angiotensinconverting enzyme (ACE) inhibitor and amitriptyline (a tricyclic
antidepressant with anticholinergic properties). Polypharmacy: postural hypotension: impaired
balance and cognition caused by temazepam, opiate analgesic and tricyclic antidepressant muscle
weakness; diuretic-induced hypokalaemia. Neurological dysfunction: brisk reflexes are suggestive
of upper motor neuron dysfunction. Common causes at this age include stroke disease, cervical
myelopathy and occasionally vitamin B12 deficiency. Falls 145 As a result of the history of falls
and the finding of cognitive impairment, a chronic subdural haematoma should also be considered.
Atrial fibrillation: poorly controlled (rapid) ventricular rate on exertion causing syncope. AF might
also be a manifestation of sick sinus syndrome, predisposing to supraventricular tachycardia or
bradycardia.
Task 3 (1.0) Poor vision: access to spectacles, ability to read or identify objects, adequacy of
lighting in the home.+
Postural hypotension: dizziness or falls when upright, with associated symptoms of faintness or
syncope, and rapid recovery when recumbent. Polypharmacy: concordance with medication
regimen and any potential to exceed the intended dosing schedule. Neurological dysfunction:
history suggestive of previous stroke or TIA. Neck arthritis: giddiness on head movement and pain
radiating to shoulders and upper limbs (cervical spondylitic radiculopathy) suggest cervical
myelopathy as a potential cause of falling. Enquire about diet, previous gastric surgery, bowel
(terminal ileal) disease or resection, anaemia and symptoms of sensory neuropathy in suspected
vitamin B12 deficiency. Fluctuating alertness: confusion or consciousness with a history of falls
and head injuries (even trivial) should highlight the possibility of subdural intracranial bleeding.
Bone pains and muscle weakness: in a housebound patient with poor diet, these point to
osteomalacia. Cardiac arrhythmia: recurrent episodes of dizziness or syncope unrelated to posture
or activity with prompt recovery suggest an intermittent cardiac rhythm disturbance.
Task 4 (1.0) Poor vision: Snellen chart to assess visual acuity, examination of eyes for common
causes of visual loss in older people (i.e. refractive disorder), cataracts, glaucoma, macular
degeneration, diabetic retinopathy. Postural hypotension: lying and standing blood pressure.
Neurological disorders: thorough central nervous system (CNS) examination essential. Focal
upper motor neuron signs suggestive of stroke or subdural haematoma. Up-going plantar
responses, consistent with cervical myelopathy; peripheral neuropathy and posterior column
dysfunction (joint position/vibration sense) suggest vitamin B12 deficiency.
Arthritis/osteomalacia: joint examination, back pain, muscle weakness.
Task 5 (2.0) The following tests are necessary: lying and standing blood pressure ECG plasma
urea, creatinine and electrolytes (U&Es) and glucose full blood count (FBC) (and vitamin
B12/folate levels if macrocytic anaemia present) radiograph of painful bones and significantly
arthritic joints vitamin D, calcium, albumin and alkaline phosphatase (ALP; for osteomalacia).
Other tests might be indicated after preliminary assessment and investigation, including: cervical
spine radiograph with option to proceed to magnetic resonance imaging (MRI) cervical spine if
cervical myelopathy is probable and surgery is a realistic option computed tomography (CT) of
the brain thyroid function.
Task 6 (2.0) Vision: occupational therapist home visit to remove environmental hazards and
improve lighting. Assess for spectacles and ophthalmic referral as needed. Postural hypotension
and polypharmacy: review the need for and doses of all medications. Neurological dysfunction:
beyond the scope of this case scenario. Osteomalacia: calcium and vitamin D supplements. Atrial
fibrillation: discuss the need for rhythm or rate control – options include cardioversion, digoxin,
beta-blockers and amiodarone. If the patient has a bradyarrhythmia, they will require assessment
for cardiac pacemaker. Owing to the risk of further falls, anticoagulation with warfarin is often not
appropriate without very careful assessment of the ongoing risks. Always involve the patient,
family and general practitioner (GP) in such discussions.

For Student
An elderly housebound woman has fallen at home on several occasions and has recently attended
A&E with a Colles’ (wrist) fracture. Her daughter is concerned about the presence of osteoporosis
and seeks your advice at the surgery about the need for calcium tablets to prevent further fractures.

Tasks:

What advice would you give the following patient?


Student’s name
For examiner

An elderly housebound woman has fallen at home on several occasions and has recently attended
A&E with a Colles’ (wrist) fracture. Her daughter is concerned about the presence of osteoporosis
and seeks your advice at the surgery about the need for calcium tablets to prevent further fractures.

What advice would you give the following patient?

Task 1 (3.5) The daughter has a valid point, but you will need to explain that to prevent further
fractures measures are needed to reduce the risk of falling in addition to a consideration of
treatment for osteoporosis.
Task 2 (3.0)The patient would benefit from a multidisciplinary assessment to identify remediable
medical illnesses that might precipitate falls (e.g. cardiovascular disease, medications, poor
eyesight, Parkinson’s disease). Advice from a physiotherapist or occupational therapist on
measures to enhance fitness and safety at home, and to secure help if further falls supervene, is
also essential.
Task 3 (3.5)If the patient is aged 75 years or older, osteoporosis can be assumed in the setting of
a low trauma fracture. Having excluded secondary causes of osteoporosis (e.g. thyrotoxicosis,
steroid therapy, multiple myeloma, hyperparathyroidism), treatment for age-associated
osteoporosis should be considered. In this setting, calcium and vitamin D supplements are helpful
and relatively safe, although it is prudent to measure the plasma calcium level before and 6–8
weeks after initiating treatment. A regular bisphosphonate is also indicated, providing the
requirements for safe and effective oral dosing can be followed. Oesophageal and peptic ulcer
disease and renal impairment will require caution with respect to bisphosphonate therapy.
Thorough discussion with the patient is essential before initiating treatment.
Final score:

For Student h

‘What is the underlying condition, and how has it come about?’ A housebound 94-year-old woman
who has epilepsy (controlled with phenytoin) presents with falls, weakness and generalized aches
and pains. Biochemical tests reveal calcium (corrected) 1.85 mmol/L (normal range 2.05–2.60
mmol/L); phosphate 0.68 mmol/L (normal range 0.8–1.45 mmol/L); albumin 32 g/L (normal range
35–48 g/L); alkaline phosphatase (ALP) 458 U/L (normal range 30–200 U/L).

Tasks:

1. What is the likely diagnosis?


2. What factors might have precipitated this condition?
3. What signs would you look for on examination?
4. How would you treat the condition?
Task 1 (3.0) The likely diagnosis is osteomalacia. Lack of exposure to sunlight, poor diet if
socially isolated, poor health, and enhanced metabolism of vitamin D as a result of treatment with
phenytoin (a liver enzyme inducer) may have precipitated the condition. Diseases causing
malabsorption and chronic renal failure can also predispose to osteomalacia.
Task 2 (2.0) You need to look for proximal muscle weakness, bony tenderness (pseudo-fractures
or Looser’s zones on radiograph), and skeletal deformities such as kyphosis and bowing of the
limbs.
Task 3 (2.0) Significant hypocalcaemia can precipitate tetany with a positive Chvostek sign
(spasm of the facial muscles on tapping over the branches of the facial nerve in front of the ear)
and Trousseau’s sign (spasm of the hand and forearm muscles after compression of the forearm).
Task 4 (3.0)The blood level should be checked for 25-hydroxy-vitamin D, and vitamin D
supplements should be given either orally or intramuscularly. An adequate calcium intake should
be provided, if necessary with oral calcium supplements (1 g/day). There should be a thorough
discussion with the patient about the risks and benefits of changing to an alternative anticonvulsant
such as sodium valproate.

Final score:

For Student
History
An 85-year-old man is admitted to hospital because of a fall in which he has sustained a mild facial
laceration. In the history it becomes evident that he has had around eight falls over the last 3
months. He says that the falls have occurred in the morning on most occasions but have
occasionally occurred in the afternoon. He does not think that he has lost consciousness although
he does remember a sensation of dizziness with the falls. He says that the falls have not been
associated with any chest pain or palpitations. He does not remember tripping or any other
mechanical trigger to the falls. He seems to return to normal within a few minutes of the fall. On
two or three occasions he has hurt his knees on falling, and on one other occasion he hit his head.
He lives alone and there have been no witnesses of any of the falls. He smokes five cigarettes a
day and does not drink. He has an occasional cough with some white sputum but he cannot
remember whether he was coughing at the time of any of the falls. He was diagnosed as having
hypertension at a routine well man clinic 4 years ago, and has been on treatment with a diuretic,
bendrofluazide and doxazosin, for this. The blood pressure has been checked in the surgery on
three or four occasions and he was told that it has been well controlled. He was found to have a
high fasting blood sugar 6 months before and had been advised a diabetic diet. There is no relevant
family history. He worked as a messenger until he retired at the age of 70 years.
Examination
He looks well. His pulse is 90/min and irregular. The blood pressure is 134/84mmHg. The heart
sounds are normal and there is nothing abnormal to find on examination of the respiratory system
or gastrointestinal system. There are no significant hypertensive changes in the fundi. In the
nervous system, there is a little loss of sensation to light touch in the toes, but no other
abnormalities.
Haemoglobin 13.8 g/dL (13.7–17.7 g/dL(
Mean corpuscular volume (MCV) 86 fL (80–99 fL)
White cell count 6.9 % 109/L (3.9–10.6 % 109/L)
Platelets 288 % 109/L (150–440 % 109/L)
Sodium 138 mmol/L (135–145 mmol/L)
Potassium 4.2 mmol/L (3.5–5.0 mmol/L)
Urea 4.6 mmol/L (2.5–6.7 mmol/L)
Creatinine 69umol/L (70–120umol/L)
Glucose 6.5 mmol/L (4.0–6.0 mmol/L) (fasting)
The ECG shows evidence of sino-atrial node disease or sick sinus syndrome.

Tasks:
What are the most likely diagnoses?
Student’s name
For examiner
History
An 85-year-old man is admitted to hospital because of a fall in which he has sustained a mild facial
laceration. In the history it becomes evident that he has had around eight falls over the last 3
months. He says that the falls have occurred in the morning on most occasions but have
occasionally occurred in the afternoon. He does not think that he has lost consciousness although
he does remember a sensation of dizziness with the falls. He says that the falls have not been
associated with any chest pain or palpitations. He does not remember tripping or any other
mechanical trigger to the falls. He seems to return to normal within a few minutes of the fall. On
two or three occasions he has hurt his knees on falling, and on one other occasion he hit his head.
He lives alone and there have been no witnesses of any of the falls. He smokes five cigarettes a
day and does not drink. He has an occasional cough with some white sputum but he cannot
remember whether he was coughing at the time of any of the falls. He was diagnosed as having
hypertension at a routine well man clinic 4 years ago, and has been on treatment with a diuretic,
bendrofluazide and doxazosin, for this. The blood pressure has been checked in the surgery on
three or four occasions and he was told that it has been well controlled. He was found to have a
high fasting blood sugar 6 months before and had been advised a diabetic diet. There is no relevant
family history. He worked as a messenger until he retired at the age of 70 years.
Examination
He looks well. His pulse is 90/min and irregular. The blood pressure is 134/84mmHg. The heart
sounds are normal and there is nothing abnormal to find on examination of the respiratory system
or gastrointestinal system. There are no significant hypertensive changes in the fundi. In the
nervous system, there is a little loss of sensation to light touch in the toes, but no other
abnormalities.
Haemoglobin 13.8 g/dL (13.7–17.7 g/dL(
Mean corpuscular volume (MCV) 86 fL (80–99 fL)
White cell count 6.9 % 109/L (3.9–10.6 % 109/L)
Platelets 288 % 109/L (150–440 % 109/L)
Sodium 138 mmol/L (135–145 mmol/L)
Potassium 4.2 mmol/L (3.5–5.0 mmol/L)
Urea 4.6 mmol/L (2.5–6.7 mmol/L)
Creatinine 69umol/L (70–120umol/L)
Glucose 6.5 mmol/L (4.0–6.0 mmol/L) (fasting)
The ECG shows evidence of sino-atrial node disease or sick sinus syndrome.
Task 1 (10.0) There are a number of possibilities to explain falls in the elderly. Some more
information in the history about the circumstances of these falls would be helpful. On further
enquiry, it emerges that the falls are most likely to occur when he gets up from bed first thing in
the morning. The afternoon events have occurred on getting up from a chair after his post-lunch
doze. These circumstances suggest a possible diagnosis of postural hypotension. This was verified
by measurements of standing and lying blood pressure – the diagnostic criteria are a drop of
15mmHg on standing for 3min. This showed a marked postural drop with blood pressure
decreasing from 134/84 to 104/68mmHg. This is most likely to be caused by the antihypertensive
treatment; both the alpha-blocker which causes vasodilatation and the diuretic might contribute.
Another possible candidate for a cause of the postural hypotension is the diabetes which could be
associated with autonomic neuropathy. In this case the diabetes is not known to have been present
for long and there is evidence of only very mild peripheral sensory neuropathy. Diabetic autonomic
neuropathy is usually associated with quite severe peripheral sensory neuropathy, with or without
motor neuropathy.
The ECG shows evidence of sino-atrial node disease or sick sinus syndrome. Clinically, it is easily
mistaken for atrial fibrillation because of the irregular rhythm and the variation in strength of beats.
The ECG shows a P-wave with each QRS complex although the P-waves change in shape and
timing. It may be associated with episodes of bradycardia and/or tachycardia which could cause
falls. This might be investigated further with a 24-h ambulatory recording of the ECG.
Coughing bouts can cause falls through cough syncope. The positive intrathoracic pressure during
coughing limits venous return to the heart. The cough is usually quite marked and he might be
expected to remember this since he gives a good account of the falls otherwise. Syncope can occur
in association with micturition. Neck movements with vertebrobasilar disease, poor eyesight and
problems with balance are other common causes of falls in the elderly. A neurological cause, such
as transient ischaemic episodes and epilepsy, is less likely with the lack of prior symptoms and the
swift recovery with clear consciousness and no neurological signs.
Another diagnosis which should be remembered in older people who fall is a subdural haematoma.
Symptoms may fluctuate, and this might be considered and ruled out with a computed tomography
(CT) scan of the brain.
The doxazosin should be stopped and another antihypertensive agent started if necessary. This
might be a beta-blocker, long-acting calcium antagonist or angiotensin convertingenzyme (ACE)
inhibitor, although all these can cause postural drops in blood pressure. His symptoms all
disappeared on withdrawal of the doxazosin. The blood pressure rose to 144/86 mmHg lying and
142/84 mmHg standing, indicating no significant postural hypotension, with reasonable blood-
pressure control.
Final score:

For Student
An 84-year-old man was found by his daughter on the floor of his apartment after failing to answer
the telephone for a day. He complained of pain in his left knee, but was otherwise confused and
unable to explain what had occurred. He was brought to the hospital by ambulance and admitted.
Past Medical/Surgical History – provided by the daughter
Myocardial infarction 10 years ago
Hypertension
Peptic ulcer disease with bleeding 20 years ago
Benign prostatic hypertrophy
Venous insufficiency
Bilateral cataract surgery 5 years ago
Left inguinal hernia repair 25 years ago
Medications:
aspirin 325 mg daily atenolol 50 mg daily one Multivitamin daily
Vitamin E 800 U daily terazosin 5 mg daily lisinopril 10 mg daily
Vitamin C 1000 mg daily Ginkgo biloba one capsule daily
Physical Examination:
In the emergency room, his physical examination revealed a thin male who appeared alert but
agitated and in moderate pain, lying on the stretcher.
His vital signs were:
Pulse 48 Weight 140 lbs
Blood pressure lying down 190/70 Height 5 ft. 9 in.
Respiratory rate 20 Temperature 99 F

Tasks:
1. Which of the following contributes to falls in the elderly?
2. Why is the systolic blood pressure elevated?
3. Why may his heart rate be slow?

Student’s name
For examiner
An 84-year-old man was found by his daughter on the floor of his apartment after failing to answer
the telephone for a day. He complained of pain in his left knee, but was otherwise confused and
unable to explain what had occurred. He was brought to the hospital by ambulance and admitted.
Past Medical/Surgical History – provided by the daughter
Myocardial infarction 10 years ago
Hypertension
Peptic ulcer disease with bleeding 20 years ago
Benign prostatic hypertrophy
Venous insufficiency
Bilateral cataract surgery 5 years ago
Left inguinal hernia repair 25 years ago
Medications:
aspirin 325 mg daily atenolol 50 mg daily one Multivitamin daily
Vitamin E 800 U daily terazosin 5 mg daily lisinopril 10 mg daily
Vitamin C 1000 mg daily Ginkgo biloba one capsule daily
Physical Examination:
In the emergency room, his physical examination revealed a thin male who appeared alert but
agitated and in moderate pain, lying on the stretcher.
His vital signs were:
Pulse 48 Weight 140 lbs
Blood pressure lying down 190/70 Height 5 ft. 9 in.
Respiratory rate 20 Temperature 99 F

Task 1 (3.5) Falls occur in 30-40% of community-dwelling elderly over the age of 65, 50% of
those over age 80, and 30-60% of nursing home residents. They are often due to a combination of
intrinsic factors (chronic illnesses, gait, strength, vision, medication effects, etc.) and extrinsic
factors (environmental and activity-related factors). Older adults should be asked at least once
yearly about falls. When evaluating falls, it is critical to obtain a careful history regarding the
events surrounding a fall in order to direct the evaluation. Multiple risk factors have been identified
and include a past history of a fall, age, cognitive impairment, female, lower extremity weakness,
balance problems, psychotropic drug use, arthritis, history of stroke, orthostatic hypotension,
dizziness, and anemia. Atenolol, a beta-blocker, can contribute to falls by causing hypotension
and/or bradyarrythmias. Terazosin, an alpha-1 adrenergic receptor antagonist (peripherally acting
alpha blocker), can cause orthostasis. Lisinopril, an ACE-inhibitor, can cause hypotension and
contribute to falls. It generally does not cause orthostasis.
Task 2 (3.0) Although cross-sectional studies have shown that blood pressure, particularly systolic
blood pressure, increases with age, however, the threshold values that define high blood pressure
are not altered for age. Blood Pressure may be elevated in response to a disease process (secondary
hypertension) or be due to essential hypertension. High blood pressure in the aged is viewed as a
disease process, rather than normal aging (as it does not occur in everyone), with an associated
increase in risk for adverse outcomes (coronary heart disease, congestive heart failure, stroke,
peripheral vascular disease, and renal disease). Although the etiology of essential hypertension is
not known, changes in peripheral vascular resistance are central to the development of
hypertension in the elderly. Resistance may be due to vascular occlusion, and/or changes in
vascular -adrenergic-mediated smooth muscle dilatation during normal aging. In addition, there
may be an expanded extracellular volume, especially in black and elderly hypertensives, which
may correspond to a low plasma renin and a high sensitivity to dietary sodium.
Task 3 (3.5) Basic sinus-node function (i.e. heart rate, sinus node recovery time, and sinoatrial
conduction time) does not change with age. Maximal heart rate declines with aging. The sinoatrial
node becomes more fibrotic with advancing age and sinus node dysfunction is common in the
elderly, but often does not cause any symptoms. When symptomatic, it is often referred to as the
“sick sinus syndrome,” which includes persistent, severe and inappropriate sinus bradycardia,
episodes of sinoatrial block, sinus arrest, or both, cessation of sinus rhythm or long pauses with
failure of subsidiary pacemakers, etc. Atenolol, a beta1-selective blocker, reduces the sinus rate at
rest and with exercise.

For Student
History
An 82-year-old man is sent up to the emergency department by his general practitioner (GP). He
is complaining of weakness and general malaise. He has complained of general pains in the
muscles and he also has some pains in the joints, particularly the elbows, wrists and knees. Three
weeks earlier, he fell and hit his leg and has some local pain related to this. He is a non-smoker
who does not drink any alcohol and has not been on any medication. Twelve years ago he had a
myocardial infarction and was put on a beta-blocker but he has not had a prescription for this in
the last 6 years. Twenty years ago he had a cholecystectomy. He used to work as a labourer until
his retirement at the age of 63 years. He lives alone in a second-floor flat. His wife died 5 years
ago. He has one son who lives in Ireland and whom he has not seen for 3 years.
Examination
He is tender over the muscles around his limb girdles and there is a little tenderness over the
elbows, wrists and knees. The mouth looks normal except that his tongue appears rather smooth.
He has no teeth and has lost his dentures. There are no other abnormalities to find in the
cardiovascular, respiratory or alimentary systems. In the legs, he has a superficial laceration on the
front of the right shin. This is oozing blood and has not healed. There is a petechial rash on some
areas of the legs. There are some larger areas of bruising on the arms and the legs which he says
have not been associated with any trauma.
Haemoglobin 10.1 g/dL (13.7–17.7 g/dL)
Mean corpuscular volume (MCV) 74 fL (80–99 f)
White cell count 7.9 % 109/L (3.9–10.6 % 109/L)
Neutrophils 6.3 % 109/L (1.8–7.7 % 109/L)
Lymphocytes 1.2 % 109/L (1.0–4.8 % 109/L)
Platelets 334 % 109/L (150–440 % 109/L)

Tasks:
1. What essential area of the history is not covered above?
2. What is the likely diagnosis?

Student’s name
For examiner
History
An 82-year-old man is sent up to the emergency department by his general practitioner (GP). He
is complaining of weakness and general malaise. He has complained of general pains in the
muscles and he also has some pains in the joints, particularly the elbows, wrists and knees. Three
weeks earlier, he fell and hit his leg and has some local pain related to this. He is a non-smoker
who does not drink any alcohol and has not been on any medication. Twelve years ago he had a
myocardial infarction and was put on a beta-blocker but he has not had a prescription for this in
the last 6 years. Twenty years ago he had a cholecystectomy. He used to work as a labourer until
his retirement at the age of 63 years. He lives alone in a second-floor flat. His wife died 5 years
ago. He has one son who lives in Ireland and whom he has not seen for 3 years.
Examination
He is tender over the muscles around his limb girdles and there is a little tenderness over the
elbows, wrists and knees. The mouth looks normal except that his tongue appears rather smooth.
He has no teeth and has lost his dentures. There are no other abnormalities to find in the
cardiovascular, respiratory or alimentary systems. In the legs, he has a superficial laceration on the
front of the right shin. This is oozing blood and has not healed. There is a petechial rash on some
areas of the legs. There are some larger areas of bruising on the arms and the legs which he says
have not been associated with any trauma.
Haemoglobin 10.1 g/dL (13.7–17.7 g/dL)
Mean corpuscular volume (MCV) 74 fL (80–99 f)
White cell count 7.9 % 109/L (3.9–10.6 % 109/L)
Neutrophils 6.3 % 109/L (1.8–7.7 % 109/L)
Lymphocytes 1.2 % 109/L (1.0–4.8 % 109/L)
Platelets 334 % 109/L (150–440 % 109/L)
Task 1 (5.0) A dietary history is an essential part of any history and is particularly important here
where a number of features point towards a possible nutritional problem. He has been a widower
for 5 years with no family support. He lives alone on a second-floor flat which may make it difficult
for him to get out. He has lost his dentures which is likely to make it difficult for him to eat.
Task 2 (5.0) He has a petechial rash which could be related to coagulation problems, but the
platelet count is normal. It would be important to examine the rash carefully to see if it is
distributed around the hair follicles. A number of the features suggest a possible diagnosis of
scurvy from vitamin C deficiency. Body stores of vitamin C are sufficient to last 2–3 months. The
rash, muscle and joint pains and tenderness, poor wound healing and microcytic anaemia are all
features of scurvy. The classic feature of bleeding from the gums would not be present in an
edentulous patient. Plasma measurements of vitamin C are difficult because of the wide range in
normal subjects. In this patient, replacement with ascorbic acid orally cleared up the symptoms
within 2 weeks. It would be important to look for other nutritional deficiencies in this situation and
to make arrangements to ensure that the situation did not recur after his discharge from hospital.

Final score:

For Student
Mr. G
Mr. G, whom we met in Case 13, has been coming regularly to your office and has agreed to most
of your recommendations. His current daily medications include amlodipine (Norvasc) 10 mg,
enalapril (Vasotec) 10 mg, glipizide (Glucotrol) 10 mg, rosiglitazone (Avandia) 8 mg, enteric-
coated aspirin 325 mg, and a multivitamin. Today, his blood pressure is 164/80 and heart rate is
72 beats per minute and regular. His weight is 186 pounds, a 4 pound reduction since his first visit
with you 6 months ago. Jugular veins are not distended, lungs are clear, and heart examination is
unremarkable. He has mild bilateral ankle edema. Significant laboratory values include HbA1c
7.5%, BUN 24 mg/dl, serum creatinine 1.8 mg/dl, total cholesterol 224 mg/dl, triglyceride 180
mg/dl, HDL 30 mg/dl, and LDL 151 mg/dl.

Tasks:
1. What criteria should be used in elderly patients to determine if blood pressure is high enough to
be treated?
2. How should you approach the management of blood pressure in Mr. G and Mrs. K?
3. What pharmacologic agents should be used in older adults with hypertension?
4. What lifestyle modifications should be recommended?

Student’s name
For examiner
Task 1 (3.0) Hypertension (HTN) is generally defined as systolic blood pressure (SBP) 140 mm
Hg or diastolic blood pressure (DBP) 90 mm Hg. Isolated systolic hypertension (ISH) is defined
as SBP 140 mm Hg or greater and DBP less than 90 mm Hg. According to recent guidelines, these
criteria apply to all adults, regardless of age (Chobanian et al., 2003), and, using these criteria,
about two-thirds of hypertensive patients between the ages of 65 and 89 have isolated systolic
hypertension. These guidelines are based on a doubling of cardiovascular mortality risk for each
20/10 increment in blood pressure over 115/75, labeling systolic pressure greater than 120 as
“prehypertension.” Before treating or counseling and possibly unnecessarily upsetting a 90-year-
old patient, it would be important to place these recommendations in perspective. SBP elevation
in late life has long been considered an inevitable consequence of advancing age, due to decreased
compliance of the arterial wall, and was considered unimportant, in contrast to diastolic
hypertension, which was thought to reflect increased peripheral vascular resistance and to be
harmful. The differing age distribution of ISH and diastolic hypertension is illustrated in Figure 6.
Longitudinal and prospective data have demonstrated clearly that SBP is a better independent
predictor for cardiovascular events than DBP, even in subjects with mildly elevated SBP. More
recently, large, double-blind, controlled studies of people 65 years of age and older with SBP 160
mm Hg demonstrated that lowering SBP significantly reduces the incidence of stroke,
cardiovascular events, and all-cause of mortality. The mean age of “elderly” subjects in most
clinical trials is about 70, and, official guidelines notwithstanding, the benefits of treatment in
elderly patients with SBP between 140 to 159 are yet to be proved.
Both of our patients satisfied the criteria for ISH. However, official criteria can only be used as a
general guide. The approach to a patient meeting the criteria of hypertension, especially those older
than 80 years with SBP between 140 and 160 without evidence of target organ damage, requires
careful consideration (see below).
Task 2 (3.0) Mr. G’s blood pressure is elevated, despite antihypertensive medications, and he has
several additional cardiac risk factors. There is ample evidence that blood pressure lowering delays
and prevents nephropathy and reduces cardiovascular events, even in patients over 65, and a target
blood pressure less than 130/85 is recommended in diabetes. Because the patient also has
proteinuria, the target might be even stricter (125/75). Although these recommendations are based
on data in patients younger than 70 years, there is currently no evidence that this target should not
be applied to Mr. G., who has compelling reasons for aggressive treatment. However, successful
treatment will depend on his ability to adhere to and tolerate the recommended regimen.
There are additional caveats to consider before treating Mrs. K, who is 90 years old. Information
on people in their nineties is sparse, and it is legitimate to ask if reducing this woman’s SBP would
benefit her in any way. Her projected life span is about 4–5 years. In two major studies of
hypertension in the elderly (see SHEP Cooperative Research Group, 1991; Staessen JAet al.,
1997), the average age of subjects was almost 20 years younger than Mrs. K. In these studies, 100
patients with ISH (SBP 160 in both studies) need to be treated with antihypertensive medications
for 2–5 years in order to achieve a reduction of stroke and major cardiovascular events by three
and five episodes, respectively. Pharmacologic treatment of hypertension can reduce the severity
of or reverse LVH, which is an independent factor for congestive heart failure. Even if Mrs. K’s
LVH were confirmed by echocardiography, any projected benefit of treatment would still be based
on the extrapolation of data from younger individuals. It will also be important to confirm whether
Mrs. K has sustained hypertension. Systolic pressure is more labile than diastolic pressure. Blood
pressure in general varies from day to day, at different times during the day, and from minute to
minute. The “white coat effect,” an elevation in blood pressure associated with measurement in
the doctor’s office, occurs in adults of all ages but the difference between office and ambulatory
blood pressure may increase with age (see Wiinberg et al., 1995). “Pseudohypertension,” or
elevated cuff pressure in the face of normal intra-arterial pressure, is another problem that should
be considered in elderly patients whose blood pressure seems hard to control. Falsely elevated cuff
pressure occurs when arteries lack distensibility or are calcified, requiring greater extrinsic
pressure before they can be compressed. The frequency of this problem is uncertain because
confirmation requires monitoring that is invasive or impractical. Nonetheless, it should be
considered, especially when patients undergoing treatment seem to develop symptoms of
hypotension in the face of normal or elevated cuff pressure. Despite these caveats, Mrs. K’s general
good health and functional status makes her an appropriate candidate for treatment. However,
vigilance is required for symptoms that might occur if blood pressure were lowered to youthful
target levels. Mrs. K preferred not to receive any antihypertensive medication, but she continued
to take coated baby aspirin and wanted to hear more about lifestyle modification. She comes to the
office periodically and her SBP has varied between 145 and 170.
Task 3 (2.0) Most antihypertensive drugs, alone or in combination, are effective in reducing SBP
and decreasing cardiovascular morbidity and mortality in elderly patients with ISH. There is no
“drug of choice” for the elderly hypertensive. Rather, the initial drug choice should be based on
the patient’s profile, comorbidity, potential side effects, and physician’s experience.
Recommendations to initiate treatment with thiazide diuretics or beta-blockers in combination with
thiazides are based on the earliest studies of ISH that used these agents and not others (SHEP
Cooperative Research 92 Case studies in geriatric medicine Group, 1991), and more recent studies
do not consistently demonstrate the benefit of one agent over another. In fact, many geriatric
patients cannot tolerate diuretics because of bladder problems (see Case 33). Specific agents
should be selected when common comorbid conditions exist for which these agents are known to
be effective – e.g. angiotensin-converting enzyme (ACE) inhibitors in diabetes or heart failure,
and beta-blockers or ACE inhibitors in myocardial infarction. Conversely, elderly patients
commonly experience certain symptoms with specific agents, such as constipation or leg edema
with calcium channel blockers (CCBs); when these effects occur, they may sometimes be
mistakenly attributed to other causes and lead to unnecessary evaluation and erroneous treatment.
Owing to pharmacokinetic changes that occur in late life (see Case 20), an increased incidence of
adverse drug reactions, and occasional heightened sensitivity to blood pressure-lowering effects,
the starting dose should be lower than, usually about one-half of, the dose for younger patients,
and the dose gradually increased if necessary. Delayed elimination of renally eliminated drugs,
such as ACE inhibitors, can sometimes be harnessed so shorter-acting agents, such as captopril,
can be given less often.
Task 3 (2.0) . Like younger adults, elderly patients with hypertension should attempt lifestyle
modification, such as weight loss if obese, exercise, smoking cessation, and avoidance of excessive
alcohol use, because the benefits of these interventions are broader than merely blood pressure
reduction. High dietary potassium intake may improve blood pressure control and an adequate
intake of potassium in foods like fruits and vegetables are recommended along with other lifestyle
modifications. Salt restriction remains somewhat controversial for elderly hypertensives, however.
Controlled studies have shown the effectiveness of dietary sodium restriction in reducing blood
pressure in hypertensive and normotensive adults. However, before recommending a salt-restricted
diet for an elderly patient, several caveats should be considered. First, evidence of benefit comes
from studies of community residing “older” adults (e.g. Johnson et al., 2001), but not in very old
or frail patients, who may already be malnourished and would possibly be harmed rather than
helped by enforced dietary restriction. Second, nonmodifiable genetic factors may determine
whether hypertensive patients are salt sensitive or salt insensitive. Third, most studies of the effects
of salt restriction employed very severe restriction of sodium, which is difficult to achieve in
clinical practice. Finally, there is a decline in renal solute conservation with age; the clinical
implications of this physiologic change with regard to salt restriction, blood pressure, or
intravascular volume are not known. Many exercises are impractical in patients with physical
limitations who are unable to increase aerobic physical activity sufficiently, if at all. In these cases,
creative interventions should be considered, including movement therapy, or a 93 Two patients
with hypertension physical therapy program tailored to the needs of the particular patient. Mrs. K
should be counseled to participate in moderate exercise, such as walking, because of overall
benefits to her well being. Mr. G requires more intensive intervention and counseling regarding
the importance of controlling blood glucose, hypertension, lipids, and weight, and might benefit
from progressive resistance training if aerobic exercises are difficult or impractical.
Final score:

For Student
Mr. G
Mr. G, whom we met in Case 13, has been coming regularly to your office and has agreed to most
of your recommendations. His current daily medications include amlodipine (Norvasc) 10 mg,
enalapril (Vasotec) 10 mg, glipizide (Glucotrol) 10 mg, rosiglitazone (Avandia) 8 mg, enteric-
coated aspirin 325 mg, and a multivitamin. Today, his blood pressure is 164/80 and heart rate is
72 beats per minute and regular. His weight is 186 pounds, a 4 pound reduction since his first visit
with you 6 months ago. Jugular veins are not distended, lungs are clear, and heart examination is
unremarkable. He has mild bilateral ankle edema. Significant laboratory values include HbA1c
7.5%, BUN 24 mg/dl, serum creatinine 1.8 mg/dl, total cholesterol 224 mg/dl, triglyceride 180
mg/dl, HDL 30 mg/dl, and LDL 151 mg/dl.

Tasks:
whether hypertensive patients are salt sensitive or salt insensitive. Third, most studies of the effects
of salt restriction employed very severe restriction of sodium, which is difficult to achieve in
clinical practice. Finally, there is a decline in renal solute conservation with age; the clinical
implications of this physiologic change with regard to salt restriction, blood pressure, or
intravascular volume are not known. Many exercises are impractical in patients with physical
limitations who are unable to increase aerobic physical activity sufficiently, if at all. In these cases,
creative interventions should be considered, including movement therapy, or a 93 Two patients
with hypertension physical therapy program tailored to the needs of the particular patient. Mrs. K
should be counseled to participate in moderate exercise, such as walking, because of overall
benefits to her well being. Mr. G requires more intensive intervention and counseling regarding
the importance of controlling blood glucose, hypertension, lipids, and weight, and might benefit
from progressive resistance training if aerobic exercises are difficult or impractical.
Final score:

For Student
Mrs. K
Mrs. K is a 90-year-old woman who has moved to the community recently. She feels well except
for pain in both knees while walking, for which she takes acetaminophen. She has also taken baby
aspirin once daily for many years. She denied a history of hypertension, diabetes, or heart disease,
and has never smoked. On physical examination, she is a slender, healthy appearing woman. Her
blood pressure is 165/80, heart rate is 78 beats per minute, and regular, respiratory rate is 12; lung,
heart and abdominal examinations are unremarkable. Examination of the lower extremities reveals
strong peripheral pulses and no edema. Blood tests including blood count, creatinine, glucose, and
thyroid-stimulating hormone (TSH) are all normal. Electrocardiogram shows left ventricular
hypertrophy (LVH) by voltage criteria, without other abnormalities. You tell her that her blood
pressure is high and she expresses doubt, saying, “I have never had high blood pressure!”

Tasks:
1. What criteria should be used in elderly patients to determine if blood pressure is high enough to
be treated?
2. How should you approach the management of blood pressure in Mr. G and Mrs. K?
3. What pharmacologic agents should be used in older adults with hypertension?
4. What lifestyle modifications should be recommended?
Student’s name
For examiner
Mrs. K
Mrs. K is a 90-year-old woman who has moved to the community recently. She feels well except
for pain in both knees while walking, for which she takes acetaminophen. She has also taken baby
aspirin once daily for many years. She denied a history of hypertension, diabetes, or heart disease,
and has never smoked. On physical examination, she is a slender, healthy appearing woman. Her
blood pressure is 165/80, heart rate is 78 beats per minute, and regular, respiratory rate is 12; lung,
heart and abdominal examinations are unremarkable. Examination of the lower extremities reveals
strong peripheral pulses and no edema. Blood tests including blood count, creatinine, glucose, and
thyroid-stimulating hormone (TSH) are all normal. Electrocardiogram shows left ventricular
hypertrophy (LVH) by voltage criteria, without other abnormalities. You tell her that her blood
pressure is high and she expresses doubt, saying, “I have never had high blood pressure!”
Task 1 (3.0) Hypertension (HTN) is generally defined as systolic blood pressure (SBP) 140 mm
Hg or diastolic blood pressure (DBP) 90 mm Hg. Isolated systolic hypertension (ISH) is defined
as SBP 140 mm Hg or greater and DBP less than 90 mm Hg. According to recent guidelines, these
criteria apply to all adults, regardless of age (Chobanian et al., 2003), and, using these criteria,
about two-thirds of hypertensive patients between the ages of 65 and 89 have isolated systolic
hypertension. These guidelines are based on a doubling of cardiovascular mortality risk for each
20/10 increment in blood pressure over 115/75, labeling systolic pressure greater than 120 as
“prehypertension.” Before treating or counseling and possibly unnecessarily upsetting a 90-year-
old patient, it would be important to place these recommendations in perspective. SBP elevation
in late life has long been considered an inevitable consequence of advancing age, due to decreased
compliance of the arterial wall, and was considered unimportant, in contrast to diastolic
hypertension, which was thought to reflect increased peripheral vascular resistance and to be
harmful. The differing age distribution of ISH and diastolic hypertension is illustrated in Figure 6.
Longitudinal and prospective data have demonstrated clearly that SBP is a better independent
predictor for cardiovascular events than DBP, even in subjects with mildly elevated SBP. More
recently, large, double-blind, controlled studies of people 65 years of age and older with SBP 160
mm Hg demonstrated that lowering SBP significantly reduces the incidence of stroke,
cardiovascular events, and all-cause of mortality. The mean age of “elderly” subjects in most
clinical trials is about 70, and, official guidelines notwithstanding, the benefits of treatment in
elderly patients with SBP between 140 to 159 are yet to be proved.
Both of our patients satisfied the criteria for ISH. However, official criteria can only be used as a
general guide. The approach to a patient meeting the criteria of hypertension, especially those older
than 80 years with SBP between 140 and 160 without evidence of target organ damage, requires
careful consideration (see below).
Task 2 (3.0) Mr. G’s blood pressure is elevated, despite antihypertensive medications, and he has
several additional cardiac risk factors. There is ample evidence that blood pressure lowering delays
and prevents nephropathy and reduces cardiovascular events, even in patients over 65, and a target
blood pressure less than 130/85 is recommended in diabetes. Because the patient also has
proteinuria, the target might be even stricter (125/75). Although these recommendations are based
on data in patients younger than 70 years, there is currently no evidence that this target should not
be applied to Mr. G., who has compelling reasons for aggressive treatment. However, successful
treatment will depend on his ability to adhere to and tolerate the recommended regimen.
There are additional caveats to consider before treating Mrs. K, who is 90 years old. Information
on people in their nineties is sparse, and it is legitimate to ask if reducing this woman’s SBP would
benefit her in any way. Her projected life span is about 4–5 years. In two major studies of
hypertension in the elderly (see SHEP Cooperative Research Group, 1991; Staessen JAet al.,
1997), the average age of subjects was almost 20 years younger than Mrs. K. In these studies, 100
patients with ISH (SBP 160 in both studies) need to be treated with antihypertensive medications
for 2–5 years in order to achieve a reduction of stroke and major cardiovascular events by three
and five episodes, respectively. Pharmacologic treatment of hypertension can reduce the severity
of or reverse LVH, which is an independent factor for congestive heart failure. Even if Mrs. K’s
LVH were confirmed by echocardiography, any projected benefit of treatment would still be based
on the extrapolation of data from younger individuals. It will also be important to confirm whether
Mrs. K has sustained hypertension. Systolic pressure is more labile than diastolic pressure. Blood
pressure in general varies from day to day, at different times during the day, and from minute to
minute. The “white coat effect,” an elevation in blood pressure associated with measurement in
the doctor’s office, occurs in adults of all ages but the difference between office and ambulatory
blood pressure may increase with age (see Wiinberg et al., 1995). “Pseudohypertension,” or
elevated cuff pressure in the face of normal intra-arterial pressure, is another problem that should
be considered in elderly patients whose blood pressure seems hard to control. Falsely elevated cuff
pressure occurs when arteries lack distensibility or are calcified, requiring greater extrinsic
pressure before they can be compressed. The frequency of this problem is uncertain because
confirmation requires monitoring that is invasive or impractical. Nonetheless, it should be
considered, especially when patients undergoing treatment seem to develop symptoms of
hypotension in the face of normal or elevated cuff pressure. Despite these caveats, Mrs. K’s general
good health and functional status makes her an appropriate candidate for treatment. However,
vigilance is required for symptoms that might occur if blood pressure were lowered to youthful
target levels. Mrs. K preferred not to receive any antihypertensive medication, but she continued
to take coated baby aspirin and wanted to hear more about lifestyle modification. She comes to the
office periodically and her SBP has varied between 145 and 170.
Task 3 (2.0) Most antihypertensive drugs, alone or in combination, are effective in reducing SBP
and decreasing cardiovascular morbidity and mortality in elderly patients with ISH. There is no
“drug of choice” for the elderly hypertensive. Rather, the initial drug choice should be based on
the patient’s profile, comorbidity, potential side effects, and physician’s experience.
Recommendations to initiate treatment with thiazide diuretics or beta-blockers in combination with
thiazides are based on the earliest studies of ISH that used these agents and not others (SHEP
Cooperative Research 92 Case studies in geriatric medicine Group, 1991), and more recent studies
do not consistently demonstrate the benefit of one agent over another. In fact, many geriatric
patients cannot tolerate diuretics because of bladder problems (see Case 33). Specific agents
should be selected when common comorbid conditions exist for which these agents are known to
be effective – e.g. angiotensin-converting enzyme (ACE) inhibitors in diabetes or heart failure,
and beta-blockers or ACE inhibitors in myocardial infarction. Conversely, elderly patients
commonly experience certain symptoms with specific agents, such as constipation or leg edema
with calcium channel blockers (CCBs); when these effects occur, they may sometimes be
mistakenly attributed to other causes and lead to unnecessary evaluation and erroneous treatment.
Owing to pharmacokinetic changes that occur in late life (see Case 20), an increased incidence of
adverse drug reactions, and occasional heightened sensitivity to blood pressure-lowering effects,
the starting dose should be lower than, usually about one-half of, the dose for younger patients,
and the dose gradually increased if necessary. Delayed elimination of renally eliminated drugs,
such as ACE inhibitors, can sometimes be harnessed so shorter-acting agents, such as captopril,
can be given less often.
Task 3 (2.0) . Like younger adults, elderly patients with hypertension should attempt lifestyle
modification, such as weight loss if obese, exercise, smoking cessation, and avoidance of excessive
alcohol use, because the benefits of these interventions are broader than merely blood pressure
reduction. High dietary potassium intake may improve blood pressure control and an adequate
intake of potassium in foods like fruits and vegetables are recommended along with other lifestyle
modifications. Salt restriction remains somewhat controversial for elderly hypertensives, however.
Controlled studies have shown the effectiveness of dietary sodium restriction in reducing blood
pressure in hypertensive and normotensive adults. However, before recommending a salt-restricted
diet for an elderly patient, several caveats should be considered. First, evidence of benefit comes
from studies of community residing “older” adults (e.g. Johnson et al., 2001), but not in very old
or frail patients, who may already be malnourished and would possibly be harmed rather than
helped by enforced dietary restriction. Second, nonmodifiable genetic factors may determine
whether hypertensive patients are salt sensitive or salt insensitive. Third, most studies of the effects
of salt restriction employed very severe restriction of sodium, which is difficult to achieve in
clinical practice. Finally, there is a decline in renal solute conservation with age; the clinical
implications of this physiologic change with regard to salt restriction, blood pressure, or
intravascular volume are not known. Many exercises are impractical in patients with physical
limitations who are unable to increase aerobic physical activity sufficiently, if at all. In these cases,
creative interventions should be considered, including movement therapy, or a 93 Two patients
with hypertension physical therapy program tailored to the needs of the particular patient. Mrs. K
should be counseled to participate in moderate exercise, such as walking, because of overall
benefits to her well being. Mr. G requires more intensive intervention and counseling regarding
the importance of controlling blood glucose, hypertension, lipids, and weight, and might benefit
from progressive resistance training if aerobic exercises are difficult or impractical.
Final score:

For Student
You have been called to see a 72-year-old man who lives alone at home. The district nurse has
become increasingly concerned about his health. The patient has been housebound for some time
but is now unable to rise from his chair. He is a large man and his left hip is very painful. There is
no history of a fall. He is unkempt and his clothes smell of urine. He is a heavy smoker, and he is
breathless and wheezy with a chronic cough. His legs are swollen and blistered. Detailed
examination is difficult but reveals signs of airflow obstruction, cyanosis, elevated JVP and
peripheral oedema. Movement at the left hip is restricted and painful. He is very reluctant to leave
his home.

Tasks:

1.What investigations would you perform?


2.What treatment options are available?
3. What are the likely causes of his immobility?
4. What can be done to improve the situation at home?
Student’s name
For examiner

You have been called to see a 72-year-old man who lives alone at home. The district nurse has
become increasingly concerned about his health. The patient has been housebound for some time
but is now unable to rise from his chair. He is a large man and his left hip is very painful. There is
no history of a fall. He is unkempt and his clothes smell of urine. He is a heavy smoker, and he is
breathless and wheezy with a chronic cough. His legs are swollen and blistered. Detailed
examination is difficult but reveals signs of airflow obstruction, cyanosis, elevated JVP and
peripheral oedema. Movement at the left hip is restricted and painful. He is very reluctant to leave
his home.
Task 1 (3.0) Basic investigations should include:
hip radiograph (essential for osteoarthritis or avascular necrosis of femoral head or fractured neck
of femur) FBC (secondary polycythaemia and infection) renal function tests (uraemia and
electrolyte imbalance) measurement of ABGs should be considered because of the signs of
respiratory failure and possible hypercapnia chest radiograph and ECG (to support the diagnosis
of chronic obstructive pulmonary disease [COPD] and cor pulmonale) peak expiratory flow rate
or spirometry before and after inhaled bronchodilator will confirm COPD and determine the degree
of reversibility.
Task 2 (3.0) Management will be difficult at home as a result of the need for radiographs and
specialist tests, and the presence of respiratory failure. Brief hospital assessment (a rapid-access
day hospital clinic or short-stay elderly care ward) is advisable to confirm the diagnoses and assess
the degree of respiratory failure. Time will be needed for rehabilitation and adaptations to the home
environment. Treatment will be along the following lines:
• analgesia for osteoarthritis (avoiding opiates and non-steroidal anti-inflammatory drugs
[NSAIDs] if possible – potential respiratory depression and fluid retention, respectively)
• bronchodilators, controlled oxygen and cautious use of diuretics for COPD and cor
pulmonale
• laxatives and enemas for faecal impaction or constipation
• in due course, consideration of hip replacement if the patient is agreeable and
cardiorespiratory status permits.
Task 3 (2.0)
The likely causes are:
• osteoarthritic hip – a hip fracture is less likely as a result of the chronicity of the symptoms
and absence of trauma
• COPD (chronic bronchitis and emphysema) and cor pulmonale
• urinary incontinence caused by immobility – almost inevitable in this situation, perhaps
exacerbated by coexisting prostatism or faecal impaction.

Task 4 (2.0) Much depends on the patient’s potential to improve and the feasibility of hip
surgery. Adaptations to the home with appropriate seating and bedding, and a review of toilet
and washing arrangements, will be necessary. The patient’s ability to climb stairs safely will
need to be assessed. He will need assistance with shopping and other household tasks. A
multidisciplinary team approach is required to resolve these various issues, including
physiotherapy, occupational therapy and social worker involvement. Early discharge from
hospital can be achieved with forward planning and use of domiciliary rehabilitation services.

Final score:

For Student

A 75-year-old man who lives in a nursing home is confined to his bed or chair as a result of a
previous left-sided stroke. The patient is a large man and his care is made even more challenging
as a result of the presence of dysphasia, dysphagia and mild cognitive impairment. The patient’s
GP visits the nursing home periodically to review his care and general condition. Give five
important areas that the GP should enquire about or assess when visiting the patient.

Tasks:

Give a brief explanation of why each of these areas needs review.

Student’s name
For examiner
A 75-year-old man who lives in a nursing home is confined to his bed or chair as a result of a
previous left-sided stroke. The patient is a large man and his care is made even more challenging
as a result of the presence of dysphasia, dysphagia and mild cognitive impairment. The patient’s
GP visits the nursing home periodically to review his care and general condition. Give five
important areas that the GP should enquire about or assess when visiting the patient.
Task 1 (3.5) The GP should enquire about or assess the following.
Skin care and pressure areas: pressure sores (especially the sacrum, hips and heels) intertrigo in
skin folds such as the groins, axillae and perineum leg ulcers complicating dependent oedema of
the lower limbs soft tissue infections.
Bladder and bowels: urinary incontinence is inevitable in this situation and will necessitate regular
toileting or containment by pads, sheath drainage or (as a last resort) urinary catheterization.
Urinary catheter care will include regular catheter changes (every 3–4 months) to prevent
encrustation, blockage and leakage. Bowel function will need review. Faecal incontinence,
constipation and faecal impaction with overflow diarrhoea may all need appropriate management.
Nutrition: obesity will exacerbate nursing difficulties. On the other hand, difficulty swallowing,
resulting in poor nutrition, might cause weight loss. Provision of food and drink of the appropriate
consistency and manoeuvres to improve swallowing will need discussion. Nutritional and vitamin
supplementation may prove necessary. Liaison with the dietitian and speech and language therapist
may be helpful.
Limb contractures/oedema: Flexion contractures of the limbs can develop over time and can be
ameliorated to some extent by appropriate positioning of the hemiplegic limb, physiotherapy, and
antispasticity drugs such as baclofen or botulinum toxin injections. Dependent oedema and DVT
can develop in an immobile lower limb. Elastic hosiery can help to prevent this. Shoulder pain can
develop as a result of incorrect handling/lifting techniques.
Respiratory complications: aspiration or hypostatic pneumonia.
Task 2 (3.0) Medication review: the need for laxatives, analgesics and hypnotics and the
appropriateness of medication for secondary stroke prevention (aspirin, a statin, antihypertensives)
will need periodic review. Social and psychological wellbeing: good-quality care should
encompass this important area. Apathy and depression are common after a disabling stroke.
Boredom and isolation can lead to difficult behaviour. Regular visits from friends and family
should be facilitated. Interest in former hobbies and pursuits should be encouraged as far as is
possible.
Task 3 (3.5) Social activities and access to radio and television are important, being mindful of
the patient’s preferences and interests. Advice from a speech and language therapist will be helpful
to maximize communication. Anticipatory care discussions should be broached so that the
patient’s wishes concerning potential future treatment interventions can be recorded. This will
require assessment of his mental capacity and, if he is willing, discussion about advance care
directives, lasting power of attorney and so on.

Final score:
For Student
A 65-year-old woman is reviewed in clinic following recent colonic surgery. Shortly after surgery
the patient developed a hot swollen leg from a deep vein thrombosis (DVT), confirmed on duplex
scanning. She has been started on anticoagulation therapy. She is upset that this developed and
wants you to explain why it occurred and what was done to prevent it.

Tasks:

How would you approach answering this patient?


Student’s name
For examiner

A 65-year-old woman is reviewed in clinic following recent colonic surgery. Shortly after surgery
the patient developed a hot swollen leg from a deep vein thrombosis (DVT), confirmed on duplex
scanning. She has been started on anticoagulation therapy. She is upset that this developed and
wants you to explain why it occurred and what was done to prevent it.
Task 1 (5.0) The essence of this case is to discuss risk reduction and distinguish this from
prevention. Deep vein thrombosis is a well-recognized problem post-surgery. The risk is higher in
people with prothrombotic states such as malignancy or a hypercoagulable syndrome (e.g. factor
V Lieden mutation) and where venous return may be compromised (pelvic surgery, lower limbs
raised intraoperatively, e.g. lithotomy position or with a pneumoperitoneum – laparoscopic
surgery).
Task 2 (5.0) To reduce the risk, the patient is given a low-molecular-weight (LMW) heparin, is
preoperatively well hydrated with intravenous (IV) fluids (especially if they have had preoperative
bowel preparation), will be wearing thromboembolic deterrent stockings and may have been asked
to stop any prothrombotic medication before surgery (e.g. tamoxifen). (LMW heparin works by
activating anti-thrombin III; it inhibits platelet aggregation and decreases the availability of
thrombin. It has a longer half-life than heparin and therefore requires only a single daily
administration. The fact that its response is much easier to predict removes the need for
monitoring.) During surgery the legs are slightly elevated to aid venous return and intermittent calf
compression can be used. Post-surgery, early mobilization is encouraged. Even with these
precautions there is still a risk of a DVT developing. The importance of making the diagnosis and
treating appropriately should be emphasized. If there has been an aberration to the normal
protocols, increasing the patient’s risk, the aberration should be admitted and a reason sought.
There is often a reason that can be explained (e.g. not giving heparin preoperatively if an epidural
is to be inserted). Did the preoperative consent discuss the risk of DVT?

Final score:
For Student

Mr. B is an 82-year-old man who is seen in clinic because of incontinence. He has a history of
Alzheimer’s type dementia, glaucoma, and dyspepsia, for which he takes donepezil, timolol eye
drops, and ranitidine, respectively. He has a history of a transurethral resection of the prostate
(TURP) at age 75 and, according to his primary care physician, his symptoms of frequency and
nocturia did not respond to a recent trial of tamsulosin.
He lives with his daughter and is ambulatory, but is dependent in all basic activities of daily living
because of dementia. She reports that his condition seems to be worsening, and he now is voiding
about every 2 hours during the day and 4 – 5 times between the hours of 9:00 p.m. and 7:00 a.m.
while he is in bed. During the day, the episodes seem to come on without warning, and he is only
able to make it to the bathroom about half the time.
About a month after his last visit, his daughter notices that over the course of a couple days his
incontinence is almost continuous: he is now only voiding in his diapers. He has also become
increasingly confused and agitated. The on-call doctor over that weekend prescribes risperidone
and lorazepam for his agitation, and diphenhydramine for insomnia, and his problems only seem
to get worse. After a fall the next day, he is taken to the emergency department, where he is found
to have a temperature of 101F, cloudy urine, and fecal impaction on examination. A post void
residual bladder volume yields 500 ml of cloudy urine. He is then admitted to the hospital.

Tasks:
1. What subtype of incontinence does the patient have in part 1? What diagnostic tools would
you use? What non-pharmacologic and drug-based treatments would be appropriate in this
case?
2. What could some possible reversible causes be for this patient’s incontinence in Part 2?
Student’s name

Task 1 (5.0) This patient has urge incontinence (UI). In older men, benign prostatic hypertrophy
can be a common cause for UI. Because Mr. B’s incontinence seems to have worsened recently, it
would be appropriate to do a urinalysis to make sure he has not developed an infection. A urinalysis
will also exclude glucosuria, a sign of diabetes (although it would be unusual for an 82-year-old
to suddenly develop diabetes). Mr. B should also have a post-void residual determination because
men are at risk for urinary retention from an enlarged prostate or urethral blockage from a stricture
that can develop after prostate surgery. Mr. B’s dementia makes prompted voiding the most
appropriate intervention to decrease his episodes of incontinence. Biofeedback and bladder
training are not helpful in patients with advanced dementia. As for pharmacologic management,
77 bladder relaxant drugs, such as tolterodine or oxybutinin, may be indicated. However, in this
patient it will be important to watch for side effects, such as delirium, dry mouth, constipation,
exacerbation of dyspnea, and increase in intraocular pressure in patients with glaucoma.
Task 2 (5.0) s affecting the ability to mobilize to the bathroom, while diphenhydramine
accomplishes this by virtue of its anticholinergic side effects. Urinary infection and fecal impaction
are also playing a role.
Final score:
For Student
An 82-year-old man with advanced Parkinson’s disease and dementia is cared for in a nursing
home. His nursing needs are usually fairly predictable. Of late, he has become more agitated than
usual. He seems uncomfortable and has started to shout and rattle the bedsides, and occasionally
he hits out at the nurses. The on-call GP has prescribed a sedative to settle the patient at night. The
patient is continuously wet with urine and, unusually for him, he is now incontinent of liquid faeces
too. He has no fever and other vital signs are normal.

Tasks:

1. What are the likely causes of urinary incontinence in this patient?


2. How should the problem be investigated?
3. What can be done to help?
Task 1 (3.5) Urinary and faecal incontinence are not uncommon in the setting of dementia and
relative immobility caused by conditions such as Parkinson’s disease. Nevertheless, regular
toileting and maintenance of a daily bowel habit by appropriate diet, simple laxatives and, when
necessary, suppositories are usually sufficient to maintain control of the situation. In this scenario,
it is very likely that faecal impaction has supervened, with overflow of liquid stool. This is a most
uncomfortable situation for the elderly man, who is unable, as a result of confusion, to draw
attention to his predicament. Urinary retention with overflow of urine is also very common in this
setting, particularly if there is pre-existing urinary outflow obstruction as a result of prostatic
enlargement. Anticholinergic drugs, sometimes used (inappropriately) in older patients with
Parkinson’s disease, and opiate-based analgesics can also precipitate this problem. Urinary
infection is not infrequent in this setting but would usually be associated with fever and offensive
urine. Infective diarrhoea would tend to produce profuse liquid stool associated with fever,
abdominal tenderness and dehydration, and there may also be other affected residents in the home.
Clostridium difficile enteritis should be considered, especially if the patient has received a
broadspectrum antibiotic.
Task 2 (3.0) A careful clinical examination (even if difficult) is essential, with particular reference
to fever, hydration, abdominal tenderness or distension, bladder enlargement (often non-tender in
chronic retention) and rectal examination (prostatic enlargement or impaction with hard stool).
Urinalysis and, if appropriate, stool culture should be done for evidence of infection. A bladder
scan should be done to assess for urinary retention. Plasma U&Es should be sent if obstructive
uropathy or dehydration is su spected.
Task 3 (3.5) Faecal impaction should be relieved by regular enemas, with attention to regular
bowel care as outlined above. Once constipation is relieved, urinary retention may resolve
spontaneously, especially if the patient is able to sit or stand to pass urine. Failing this, temporary
urinary catheterization (with measurement of residual urine to confirm the diagnosis) should be
performed until the bowel function has normalized. Anticholinergic and opiate-based medication
should be withdrawn whenever possible. Treatment for UTI should be guided by antibiotic
therapy. Infectious diarrhoea will require ‘barrier nursing’ precautions to prevent the spread of
infection. Clostridium difficile enteritis will require oral metronidazole or vancomycin.

Final score:
For Student

An elderly woman with hypertension is attending the surgery for regular assessment of her blood
pressure. On this occasion her daughter attends as well and mentions that the family is very
concerned about the patient’s apparent incontinence of urine. The daughter goes on to say that her
mother’s house and clothing have now developed a constant smell of urine. On direct questioning,
the patient makes light of this problem. She seems sensible.

Tasks:

How would you proceed?


Student’s name
For examiner
An elderly woman with hypertension is attending the surgery for regular assessment of her blood
pressure. On this occasion her daughter attends as well and mentions that the family is very
concerned about the patient’s apparent incontinence of urine. The daughter goes on to say that her
mother’s house and clothing have now developed a constant smell of urine. On direct questioning,
the patient makes light of this problem. She seems sensible.
Task 1 (3.5) Urinary incontinence is a common problem in old age and is often unreported. The
reasons for this reluctance to seek medical advice are multifactorial and include embarrassment
and a belief that nothing can be done or that it is an inevitable consequence of ageing. Health-care
professionals may fail to enquire about the symptom or, if it is mentioned, may feel unable to
intervene in an effective manner.
Task 2 (3.0) The first step is to secure the patient’s confidence and trust. An approach by the
practice nurse may be a more successful first step to gain an initial history and preliminary
assessment. It will be important for the nurse or doctor to emphasize that assessment is relatively
simple and that an effective treatment is very likely. Even if cure is not feasible, improvement or
containment of symptoms is certainly possible.
Task 3 (3.5) The initial assessment should include a thorough history and examination (note rectal
and vaginal examinations), paying particular attention to medications and any gynaecological or
neurological symptoms and signs. Urinalysis, urine culture and bladder scan (to assess bladder
emptying and residual volume) are also essential steps in the assessment. Most assessments can be
completed in a general practice setting, and measures can be instituted to improve the common
conditions of urge and stress incontinence. The advice of a continence nurse specialist is useful for
the treatment of complex cases and where a more detailed knowledge of containment devices and
pads is required.
Final score:

For Student
An 86-year-old woman is distressed by persistent incontinence of urine.

Tasks:
Outline the key steps to establishing the cause of this problem in a primary care setting.
Student’s name
For examiner

An 86-year-old woman is distressed by persistent incontinence of urine.

Task 1 (3.5) The key steps include a full history focusing on:
• precipitating factors
• pattern of voiding, including assessment of frequency and volume
• previous and current medical and surgical illnesses, including obstetric procedures and
problems
• drug history, especially diuretics, anticholinergics, sedatives and opiates
• assessment of functional status and toileting arrangements
• corroborative details from carer or relative.
Task 2 (3.0) A full examination should be done, focusing on:
• abdomen – retention of urine, pelvic masses
• rectal examination – constipation, tumour, anal tone
• vaginal examination – prolapse, tumour, atrophic changes
• full neurological assessment – search for evidence of cognitive dysfunction and other CNS
diseases, e.g. stroke, Parkinson’s disease, spinal cord disease, autonomic dysfunction
• locomotor examination.
Task 3 (3.5) Simple, focused investigations in response to the findings on history and examination
include:
• urinalysis
• MSU
• measurement of residual volume (by bladder scan or catheterization)
• capillary glucose
• plasma urea, creatinine and electrolytes
• plain radiograph or ultrasound scan of kidneys, ureters and bladder (for renal/bladder stone)
• urodynamic assessment if the above investigations fail to elucidate the cause.

Final score:
For Student
History
Mrs. L, a 75-year-old widow, came to your office after being discharged from the hospital, where
she underwent surgery for a fracture of her right shoulder. Mrs. L has been under your care for
several years and has been treated for hypertension, osteoarthritis of both knees, and obesity. She
had a stroke 4 years ago but the deficit resolved. She has no history of diabetes or glaucoma. Her
hypertension had been well controlled with daily hydrochlorothiazide 25 mg and atenonol 50 mg.
Because she does not tolerate nonsteroidal antiinflammatory agents, she takes acetaminophen for
her knee pain but still has pain when she walks and sometimes uses a cane. Other medications
include enteric-coated aspirin and a multivitamin.
Mrs. L explains that, on the night of the fracture, she woke up to urinate around midnight, and then
fell and broke her shoulder. She related her fall to drinking wine that night with a friend, which
had made her a little drowsier than usual when she got up at midnight. She drinks alcohol only
occasionally, and has not had trouble before. The conversation reminded Mrs. L that she
experienced frequent nocturnal urination during the hospitalization and on several occasions was
unable to get to the toilet on time and became incontinent. When questioned, she admits that she
has had urinary frequency for several years but managed it by avoiding beverages before sleep or
before leaving her house. She also avoids going out for long periods during the day, and, whenever
she returns from her brief excursions, she develops urinary urgency “as soon as the key goes into
the lock.” She has occasionally experienced leakage when sneezing, standing, or coughing, but
this most commonly occurs when she is trying to hold her urine during one of her “urgent”
episodes. Still, she did not view her urinary pattern as a big problem until her recent hospitalization.
Mrs. L last visited her gynecologist 1 year ago. She has no cystocele, rectocele, or uterine prolapse.
She denies dysuria, fever, or constipation.

Tasks:
1, What factors contributed to this patient’s urinary incontinence?

2. How should her problem be approached?


3. What nonpharmacologic approaches could be of benefit to this patient?
Student’s name
For examiner

History
Mrs. L, a 75-year-old widow, came to your office after being discharged from the hospital, where
she underwent surgery for a fracture of her right shoulder. Mrs. L has been under your care for
several years and has been treated for hypertension, osteoarthritis of both knees, and obesity. She
had a stroke 4 years ago but the deficit resolved. She has no history of diabetes or glaucoma. Her
hypertension had been well controlled with daily hydrochlorothiazide 25 mg and atenonol 50 mg.
Because she does not tolerate nonsteroidal antiinflammatory agents, she takes acetaminophen for
her knee pain but still has pain when she walks and sometimes uses a cane. Other medications
include enteric-coated aspirin and a multivitamin.
Mrs. L explains that, on the night of the fracture, she woke up to urinate around midnight, and then
fell and broke her shoulder. She related her fall to drinking wine that night with a friend, which
had made her a little drowsier than usual when she got up at midnight. She drinks alcohol only
occasionally, and has not had trouble before. The conversation reminded Mrs. L that she
experienced frequent nocturnal urination during the hospitalization and on several occasions was
unable to get to the toilet on time and became incontinent. When questioned, she admits that she
has had urinary frequency for several years but managed it by avoiding beverages before sleep or
before leaving her house. She also avoids going out for long periods during the day, and, whenever
she returns from her brief excursions, she develops urinary urgency “as soon as the key goes into
the lock.” She has occasionally experienced leakage when sneezing, standing, or coughing, but
this most commonly occurs when she is trying to hold her urine during one of her “urgent”
episodes. Still, she did not view her urinary pattern as a big problem until her recent hospitalization.
Mrs. L last visited her gynecologist 1 year ago. She has no cystocele, rectocele, or uterine prolapse.
She denies dysuria, fever, or constipation.
Task 1 (3.5) The patient’s chronic urinary urgency suggests that she suffers from detrusor
instability, but her problem may be multifactorial. She has been taking diuretics, which could be
exacerbating her problem. Although she has found her bladder problems annoying, she coped with
them until she became incontinent in the hospital. Urinary “urge incontinence” is a typical
presentation of detrusor instability. The diagnosis of detrusor instability (also referred to as
“overactive,” “unstable,” “hyperreflexic,” “spastic,” or “uninhibited neurogenic” bladder) is
generally apparent from the history alone. Typically, the affected patient reports frequent
involuntary contractions or severe urgency at a relatively lower bladder volume. Detrusor
instability is the most common cause of urinary incontinence in elderly men and women. It can be
caused by a neurologic condition, such as dementia or stroke, which releases the brainstem
detrusor-reflex from cerebrocortical inhibition. In most cases, however, no specific neurologic
illness is identified, and incontinent elderly patients who develop stroke or other brain lesions may
previously have had detrusor instability. In the setting of detrusor instability, bladder irritation due
to infection, bladder tumor, or stone can worsen existing urgency and frequency. In Mrs. L’s case,
the long history of bladder symptoms, and the absence of fever and dysuria, make urinary tract
infection a less likely cause of acute incontinence. Mrs. L also has occasional stress incontinence,
which can coexist with detrusor instability in women, and which may present in the
perimenopausal years and even earlier. This “true” stress incontinence (in contrast to stress
incontinence in the presence of urinary retention) is characterized by leakage of urine in association
with sudden increased intra-abdominal pressure during coughing, sneezing, laughing, or, in severe
cases, merely standing up. It is due to insufficiency of the internal urethral sphincter or pelvic floor
weakness. The syndrome has been attributed to estrogen deficiency and childbirth, but variable
clinical response to estrogen replacement, and existence of the problem in nulliparous women,
challenge those explanations or suggest that other factors are involved.
Overflow urinary incontinence occurs in the setting of significant urinary retention. Incontinence
may be precipitated by increased intra-abdominal pressure, causing a reflex micturition contraction
and loss of a small amount of urine. Persistent urinary retention is uncommon in women and, on
examination, Mrs. L’s bladder was not palpable. Urinary retention, its causes, and management
are discussed in detail in Case 34. Mrs. L’s “acute” incontinence was partially functional in nature
and was precipitated by circumstances associated with hospitalization. Functional urinary
incontinence (also called “pseudoincontinence”) occurs when the patient is unable to reach the
toilet on time because of physical limitations, environmental barriers, or a pharmacologic effect,
such as sedation. In Mrs. L’s case, ambulatory problems due to arthritis, now complicated by
problems with her right arm and deconditioning during hospitalization, prevented her from
toileting quickly enough and she became incontinent. In addition to her ambulatory limitations,
detrusor instability, diuretics, and alcohol (with its sedating as well as diuretic properties) were all
important factors in her fall.
Task 2 (3.0) Like many patients, Mrs. L’s long-standing urinary frequency did not prompt her to
seek medical attention, and her physicians did not inquire. In general, it is important to inquire
routinely about bladder problems in older adults. Because Mrs. L recently underwent surgery and
probably had an indwelling catheter, urinalysis and culture should now be performed to rule out a
hospital-acquired urinary tract infection as a factor in her “acute” incontinence. However,
asymptomatic bacteriuria often exists in late life (see Case 34) and treatment of the infection does
not reverse the incontinence. In order to address her ongoing bladder problem, the diuretic could
be replaced by an antihypertensive agent that would not exacerbate her bladder problems. It is best
to avoid certain antihypertensive agents, such as alpha-blockers, which can relax the urethral
sphincter and are, in fact, commonly given to promote voiding in benign prostatic hyperplasia.
Calcium channel blockers, though generally well tolerated, have occasionally been reported to
cause urinary retention, probably because detrusor contractions are dependent on calcium
channels. Other medications that can be problematic are sedatives, which can cause confusion and
missed bladder cues. Medications that relax the detrusor muscle, such as oxybutynin (Ditropan) or
tolterodine (Detrol), can sometimes ameliorate detrusor instability but can cause dry mouth, visual
blurring, and other anticholinergic effects. Systemic and topical estrogen are frequently given for
urinary stress incontinence but clinical trials have not consistently supported their benefit, even in
younger women. Mrs. L was given enalapril instead of hydrochlorothiazide. Her urinalysis was
normal, and on follow up 1 week later, and her blood pressure was still under control. Her urinary
frequency seemed diminished, but she still had to urinate two to three times per night and continued
to have frequency during the day. Oxybutynin was instituted at a very small dose, and she was
instructed to take the medication only at specific times, such as prior to a social activity, a physical
therapy session, or at bedtime. This reduced her urinary frequency somewhat.
Task 3 (3.5) Removal of environmental hazards and a good night light can make it easier to get to
the bathroom safely and on time. A bedside commode can make night-time toileting easier and
can help to prevent functional incontinence. These interventions would be particularly important
in this patient who fractured her arm on the way to the bathroom at night. Pelvic muscle exercises
are helpful for patients with stress incontinence and for some with urge incontinence. The patient
first is taught to identify the pelvic muscles that will be exercised, then to try to stop the stream in
the middle of urination, let it resume, and then stop the stream again. The exercise consists of 10-
second contractions followed by 10-second relaxations, and the exercise is repeated 15 times
approximately three times a day. The ideal patient is cognitively intact, ambulatory, and able to
perform this exercise correctly, so it is not suitable for many frail older patients. Biofeedback
instruments are sometimes used to help the patient learn to identify pelvic muscles and to master
the technique of exercising pelvic muscles selectively while keeping abdominal muscle relaxed.
Bladder training is another approach that could be helpful. This consists of the patient or caregiver
observing and recording the patient’s micturition needs, and toileting at the longest possible
interval (usually 30 minutes to 2 hours) to keep her dry. If continence is maintained for 48 hours,
the interval can be lengthened. This method is repeated until a reasonable goal is achieved, such
as 4 hours of continence. Patients with urge incontinence are taught to employ “urge strategies,”
which are adaptive responses to the sensation of urgency. These include distraction, relaxation of
the entire body, or contracting pelvic muscles instead of rushing to the toilet. After urgency
subsides, the patient proceeds to the toilet at a normal pace. For patients with dementia, who will
be unable to use these strategies or for others who cannot toilet on their own, a caregiver observes
the patient’s voiding patterns and maintains a regular toileting schedule in accordance with the
observed pattern. Adult incontinence garments and pads are commercially available or can be
improvised in the home or hospital and can be used as a backup for “accidents” to maintain dryness
and to give the patient confidence to participate in social activities.

For Student
An 85-year-old man with a history of benign prostatic hyperplasia and no history of prostate cancer
or prostate procedures comes into your outpatient clinic complaining of chronic incontinence. He
says that he is frustrated because he is nearly always wet, and doesn’t notice when he is about to
become incontinent. When he does go to the bathroom to void, he has difficulty initiating a urinary
stream, and has some dribbling afterward.

Tasks:
What type of incontinence does this patient most likely have?
How it should be managed?

Task 1 (5.0)
The answer is (c). He suffers from overflow incontinence secondary to chronic urinary obstruction.
If it were urge incontinence, he would be receiving some warning that he needs to void. Because
it is chronic, it is less likely to be caused by something new, such as a urinary tract infection. He
is fully able to go to the bathroom by himself, so this is not functional incontinence.
Task 2 (5.0) treatment of undelying disease.
Marked prostatic enlargement Gross enlargement of the prostate on digital
examination; prominent induration or asymmetry of
the lobes.
Final score:
For Student

A 70-year-old woman with a history of congestive heart failure and diabetes mellitus is seen in
your clinic because of frequent episodes of incontinence. She says that these episodes happen
without warning, usually in the context of coughing or sneezing.

Tasks:
1. What is the possible mechanism of patient’s incontinence
2. Which of these interventions is most likely to help with her incontinence?
Task 1 (5.0) Task 2 (5.0) This patient has stress incontinence, which has been shown to be more
responsive to behavioral interventions, like pelvic muscle exercises, than drug treatments such as
topical estrogen. Estrogen, in fact, has shown little efficacy in clinical trials. Tolterodine is helpful
in urge, not stress incontinence. Alpha blockers such as tamsulosin have a very limited role in
women. Augmentation cystoplasty is a surgical treatment for patients with diminished bladder
capacity.
Student’s name
For examiner

Final score:

For Student
You are the new primary care physician for a 78 year-old-man with advanced Alzheimer’s disease
who lives in a nursing home. Other than his dementia, he is relatively healthy, and is able to
participate in group activities. His nurse is very concerned about his chronic incontinence. She
states that, since he has been admitted, he has chronically been wetting his diaper. She is concerned
that she has noticed increasing breakdown in his perineal skin, and would like a treatment for his
incontinence.

Tasks:
1. What is the possible mechanism of patient’s incontinence?
2. What would be the initial best treatment for this patient?
Task 1 (5.0) Task 2 (5.0) The answer is (d). In a patient with functional incontinence and cognitive
impairment, prompted voiding has been shown to reduce the severity of the problem by half
(Ouslander JG et al. JAMA 1996; 273:1366-70). Pelvic muscle exercises, anticholinergics, and
alpha blockers don’t address the issue. While a urinary catheter would address the problem of skin
breakdown and eliminate the incontinence, it would be associated with a high risk of urinary
infections and falls.

Final score:
For Student
History
An 84-year-old retired professor was admitted to the coronary care unit of an acute care hospital
with tachycardia and congestive heart failure. He had a history of chronic constipation for which
he had taken various sennacontaining preparations for many years. His most recent bowel regimen
also included psyllium hydrophilic mucilloid (Metamucil) twice daily, docusate sodium (Colace)
300 mg daily, daily prune juice, and milk of magnesia 30 cc as needed. He had no known history
of urinary difficulties.
Physical findings included atrial fibrillation with a ventricular rate of 150, and bibasilar rales.
Treatment consisted of enalapril and furosemide, diltiazem for rate control, and morphine for
dyspnea. A low salt diet was ordered and Colace 100 mg daily was prescribed for constipation.
The patient was kept on bed rest.
On the second hospital day, the patient complained that he was “not being given enough laxatives,”
and his diet orders were modified to include 10 g of bran with his breakfast cereal, and two high-
fiber cookies (5 g of fiber each) for dessert at lunch and dinner. At this time, diphenhydramine
(Benadryl) was added for insomnia.
By the third hospital day, his cardiac status had improved but he began to complain of an inability
to void, whereupon it was noted that he had not passed urine for at least 8 hours, although he had
been eating and drinking normally. A Foley catheter was inserted and 550 cc of clear urine was
passed. The catheter remained in place while cardiac workup was completed over the next few
days.
On the fifth hospital day, the patient developed a temperature of 101 oF (oral) and urinalysis
revealed many bacteria and eight to ten white blood cells per high power field. Urine and blood
cultures were sent to the laboratory and ciprofloxacin was begun. The patient began to complain
that the catheter was “annoying” him and that it was “probably to blame for the infection in the
first place.” He demanded that it be removed. The catheter was removed but, when the patient was
unable to pass urine 8 hours later, it was reinserted.

Tasks:
1. What factors have contributed to this patient’s problem?
2. How should this problem be managed?
3. What factors contributed to the urinary tract infection?
4. What additional factors contributed to this patient’s dilemma?

Task 1 (3.0). The patient’s constipation, imposition of bed rest, medications, and, as an elderly
male, his likelihood of underlying benign prostatic hyperplasia (BPH; see Figure 14) have all
contributed to acute urinary retention. Rectal examination revealed a hard mass that had resulted
in fecal impaction and accumulation of abundant feces in the rectum, compressing the bladder and
resulting in urinary retention. Stool impaction is implicated as a cause of urinary retention in up to
10% of older hospitalized patients. Morphine, diltiazem, and diphenhydramine are likely to have
contributed to this patient’s fecal impaction by slowing bowel transit time. The anticholinergic
activity of morphine and diphenhydramine also contributed to urinary retention directly, by
inhibiting the detrusor muscle of the bladder, which contracts in response to cholinergic
stimulation. Common offenders include tricyclic antidepressants, first-generation antihistamines,
opioids, and gastrointestinal antispasmodics. Antispasmodics such as oxybutinin (Ditropan) and
tamsulosin (Detrol), which are used to treat detrusor instability (see Case 33), may result in
unwanted urinary retention. The probability that this patient had some degree of BPH increased
the likelihood that he would develop acute urinary retention. The prevalence of BPH increases
with age and, by age 80, approximately 80% of men have pathologic evidence of BPH. According
to a large prospective study, men aged 70–79 years have a one in ten chance of developing acute
urinary retention in the next 5 years, and the risk for men in their eighties is almost one in three
Task 2 (3.0). Fecal impaction can be treated with enemas, but, when a hard mass is felt on digital
rectal examination, manual disimpaction should be initiated first. This patient was treated with
manual disimpaction, after which he expelled soft feces spontaneously. Appropriate treatment
would also include discontinuation, when possible, of medications that interfere with bladder and
bowel function – in this case, morphine and diphenhydramine. Diltiazem, which is very
constipating, should be replaced if possible with a beta-blocker. Enemas can be given if necessary,
and follow up should include bathroom or commode privileges, physical activity as soon as
possible, and removal of the catheter. Antibiotics are not always necessary in catheter-induced
urinary tract infections, once the catheter has been removed, as most catheter-induced infections
are asymptomatic and bacteriuria may resolve on its own. Acute urinary retention is extremely
common in older men when they are hospitalized. Bowel function needs to be monitored on a daily
basis in patients at risk, and constipation treated right away in order to prevent fecal impaction.
Early mobilization, avoidance of dehydration, avoidance of constipating medications, and
avoidance of anticholinergic medications are other maneuvers that need to be employed. Male
patients should be assisted to get out of bed and stand to urinate if possible, or, if they are unable
to use a urinal or urinate effectively with one, a bedside commode may help. Likewise, bowel
movements are easier if the patient can be seated on the toilet, in the privacy of the bathroom.
Bladder catheters should be discontinued as soon as minute-to-minute measurement of urinary
output is no longer necessary, in order to avoid infection. Patients who have no history of
symptomatic urinary retention prior to hospitalization generally are again able to urinate if
precipitating factors are rectified. However, it is not unusual for an elderly man to be unaware of
a high urine residual volume, and urinary retention sometimes stubbornly persists. In these cases,
tamsulosin (Flomax) should be considered. This alpha-blocking agent selectively inhibits alpha-
1a adrenergic receptors, reducing smooth muscle contractions, reducing intraurethral pressure, and
increasing urine flow. Because tamsulosin has a greater specificity for the receptors in the bladder
and prostate, it does not lower systemic blood pressure and is an appropriate first-line treatment
for obstructive symptoms in BPH. Alternatively, a nonselective alpha-blocker such as terazosin
(Hytrin) can be given in patients requiring treatment for hypertension.
Task 3 (2.0). The indwelling catheter, acute urinary retention, and likelihood that the patient had
underlying BPH probably all contributed to the urinary tract infection. The incidence of catheter-
related urinary tract infections increases rapidly with the duration of catheterization at a rate of
about 10% per day. In addition, this patient had developed urinary retention prior to catheterization
and was already predisposed to infection. With BPH, this patient probably would have had an
elevated postvoid residual urine (see Case 33), increasing his risk of stasis and bacteriuria. Loss of
barrier defenses is an additional risk factor for urinary tract infections and bacterial colonization
in elderly patients. The uroepithelium appears to be less effective in providing a barrier to
infection. Changes in surface glycosaminoglycans and fibronectins contribute to adherence of
bacteria. In elderly women, predisposing factors include increased adherence of vaginal
uropathogens to uroepithelial cells, cystocele, which can increase postvoid residual volume, and
estrogen deficiency, which promotes colonization of the vagina with urinary pathogens.
Task 4 (2.0). The patient was known to suffer from chronic constipation. Intestinal transit time is
prolonged with age, but constipation is not a universal complaint. Chronic use of such laxatives
may permanently damage the electrical system of the colon. Irritant laxatives such as senna, and
possibly others, may directly affect the myenteric plexus. The result is “cathartic bowel,” an
overdistended colon with loss of haustrations and poor motility. Other irritant laxatives that may
cause cathartic bowel include 229 Urinary retention castor oil, cascara, aloe, bisacodyl, and
phenolphthalein. Prunes and prune juice contain a phenolphthalein derivative but have not been
reported to cause cathartic bowel. This patient’s bowel movements occurred to his satisfaction
only with a strict bowel regimen. This regimen was not given to him in the hospital, possibly
because it was viewed by the staff as an eccentric and excessive use of laxatives. Also, medications
are not always part of the hospital formulary and alternatives are not always sought for what might
be perceived as a “nonessential” medication. The sudden imposition of bed rest probably reduced
his bowel motility further. The diuretic and salt restriction, combined with age-related decline in
renal concentrating ability, is likely to have caused a relative dehydration and reduced water
content of feces. Although bran has been shown to reduce intestinal transit time in elderly
hospitalized patients, bran supplementation can sometimes form a bulky mass and cause fecal
impaction, especially when the water content of the diet is not concurrently increased, or if the
patient is bedridden. The difficulty of supine defecation and the lack of privacy are also likely to
be contributors to this patient’s dilemma, but these modifiable risk factors are often overlooked in
the hospital setting.
For Student
History
An 85-year-old woman is brought to the emergency department by her home health aide because
of shortness of breath and worsening cough for 1 week. She has a long history of hypertension,
diabetes, chronic obstructive pulmonary disease (COPD), bronchiectasis, osteoporosis, and
osteoarthritis. She had smoked heavily all her adult life but quit 20 years ago. Over the last 2 years,
she has had several admissions for COPD exacerbation, which responded to antibiotics and
corticosteroids.
The patient adheres carefully to her regimen of medications, which include inhaled corticosteroids
and bronchodilators, lisinopril, rosiglitazone, calcium and vitamin D, and acetaminophen. Her
blood pressure and blood sugar have been under good control. She lives by herself and is assisted
by a home health aide 8 hours a day.
In the emergency room, her blood pressure is 170/90, pulse is 88 and regular, respiratory rate is 28
per minute, and temperature is 98 ◦F. Lung examination reveals basilar crackles (unchanged from
previous examinations), diffuse rhonchi, and wheezing. No peripheral edema is noted. The
electrocardiogram (EKG) reveals left ventricular hypertrophy (LVH). Chest X-ray shows poor
inspiration, increased vascular markings, and a retrocardiac infiltrate. She is admitted and treated
with intravenous methylprednisone, antibiotics, and nebulized bronchodilators. She feels better in
a few hours.
The next morning, to your surprise, you find the patient’s respiratory status has worsened. She is
sitting upright in bed, has labored breathing, and wheezing can be heard without the stethoscope.
A repeat chest X-ray is performed. A repeat chest X-ray showed small bilateral pleural effusions
and pulmonary vascular congestion, in addition to the previous finding of a retrocardiac infiltrate

Tasks:
1. What is the cause of the patient’s worsening condition?
2. What is the pathophysiology? How can the diagnosis be confirmed?
3. What factors could have contributed to this patient’s problem?
4. How should this condition be treated?
Task 1 (3.0) When an elderly patient presents with cough, dyspnea, or wheezing, congestive heart
failure (CHF) should always be in the differential diagnosis, even when there is a history of COPD.
This is especially important when the patient has a history of hypertension, diabetes, or ischemic
heart disease. This patient’s medical history made COPD exacerbation the major consideration on
admission. However, despite aggressive treatment, her wheezing and dyspnea eventually
worsened. Wheezing is a common finding in elderly patients with CHF, even in those without a
known history of bronchospastic disease. Patients with this “cardiac asthma” have a
bronchospastic response to methacholine, compared with heart failure patients who do not have
wheezing. A repeat chest X-ray showed small bilateral pleural effusions and pulmonary vascular
congestion, in addition to the previous finding of a retrocardiac infiltrate. Intravenous furosemide
and morphine were given and the patient improved. The dose of lisinopril was increased to 10 mg
daily, with a plan to increase further, as tolerated. Repeat EKG showed no changes and cardiac
enzymes were normal. Thyroid function tests and repeat complete blood count (CBC) were
ordered, and an echocardiogram was performed.
Task 2 (3.0) The patient’s echocardiogram revealed LVH, normal left ventricular (LV) function
with an ejection fraction (EF) of 65%, and mild pulmonary hypertension. CHF with preserved left
ventricular function suggests the patient has diastolic dysfunction – an important cause of CHF in
the elderly. Heart failure can be associated with either reduced cardiac output (LV systolic
dysfunction) or with normal cardiac output. If the maintenance of adequate cardiac output requires
a higher than normal LV filling pressure, it is referred to as diastolic dysfunction. The patient may
be asymptomatic or may experience only reduced exercise tolerance, or may have overt heart
failure that is clinically identical to LV failure. Among patients with CHF, diastolic dysfunction
accounts for fewer than 103 Wheezing 10% of cases under the age of 65, but the proportion
increases with advancing age, accounting for more than 50% of cases after age 75. Many factors
may affect diastolic function, the most important being LV relaxation, LV stiffness, and left atrial
function. LV relaxation, which occurs during early diastole, is an energy-dependent process and
can be readily affected by ischemia. LV diastolic stiffness or compliance is affected by myocardial
fiber distensibility, connective tissue elasticity, chamber size, wall thickness, and the condition of
the pericardium. Left atrial contraction, which occurs in late diastole, accounts for less than 20%
of filling volume in young, healthy persons, whereas, in patients with early diastolic dysfunction,
the left atrium increases contractility to compensate for LV diastolic dysfunction and can
contribute up to 50% of the filling volume. Diastolic dysfunction can be caused by impaired
energy-dependent ventricular relaxation, which occurs in ischemic heart disease, hypertrophy,
tachycardia, and increased afterload such as in aortic stenosis, or it can be due to reduced passive
elastic properties, which occur in hypertension with LVH or if there are increased myocardial
connective tissue components, such as fibrosis, diabetes, or infiltrative diseases such as
amyloidosis. Both mechanisms exist to a limited degree in normal aging. Diastolic dysfunction is
more common in women than in men and is the cause of nearly two-thirds of all CHF cases in
women over the age of 80. The reason for the gender difference is not clear, but it may be related
to the greater frequency of systolic heart failure in the male cohort. Accurate diagnosis of diastolic
dysfunction requires simultaneous measurements of LV pressures and volumes to create pressure–
volume curves, using cardiac catheterization. This is invasive and unpractical. Clinically, the
diagnosis of diastolic heart failure relies on clinical criteria for CHF with documentation of
preserved LV function by Doppler echocardiogram (i.e. EF >45%), and absence of other
conditions that also cause CHF with normal LV systolic function, such as anemia and
thyrotoxicosis. This patient’s thyroid function tests and CBC were normal.
Task 3 (2.0) Long-standing hypertension, diabetes, and possible coronary artery disease, as well
as aging, probably all contributed to this patient’s diastolic dysfunction. Thiazolidinediones, such
as rosiglitazone, may cause fluid retention and precipitate overt CHF in patients with underlying
cardiac dysfunction. These drugs are believed to produce peripheral vasodilatation, which leads to
decreased mean arterial pressure and consequent renal retention of sodium and water. Recently, in
vitro evidence of pulmonary endothelial cell permeability has been reported (see Idris et al., 2003),
but more research is needed to confirm a cellular mechanism. The exacerbation of COPD and the
overnight intravenous fluid infusion might also have contributed to the rapid worsening of this
patient’s CHF. After alleviation of pulmonary congestion with diuretics, the dose of lisinopril was
further increased to 20 mg and the patient tolerated it well. Her weight decreased by 104 Case
studies in geriatric medicine 6 pounds over 4 days. Her pneumonia and COPD improved and
corticosteroids were tapered. Rosiglitazone was discontinued, glipizide was instituted, and the
patient was discharged home.
Task 4 (2.0) Treatment of underlying or exacerbating conditions, such as hypertension,
myocardial ischemia, and tachyarrhythmias, is important. In contrast to systolic heart failure,
virtually no randomized controlled trials have been conducted of the treatment of diastolic heart
failure. Clinical experience and limited evidence indicate that most treatments for systolic heart
failure can also benefit patients with diastolic heart failure. Diuretics are useful for acute
exacerbation of pulmonary congestion and fluid retention, such as in this case. However, since
patients with diastolic dysfunction depend on an adequate preload to maintain normal cardiac
output, overuse of diuretics should be avoided. There is some evidence that angiotensin-converting
enzyme (ACE) inhibitors and angiotensin receptor blockers (ARBs) improve diastolic function. A
possible mechanism might be their ability to affect beneficially the myocardial remodeling process
in CHF. Beta-blockers may also be beneficial for patients with diastolic dysfunction. In addition
to their antihypertensive and anti-ischemic properties, they improve diastolic filling by reducing
heart rate in patients with tachycardia. Digoxin is not usually recommended in patients with CHF
associated with diastolic dysfunction, but it has been used successfully and safely to control heart
rate in patients with atrial fibrillation coexisting with diastolic dysfunction.

Case 1:
Mrs DM Scenario
This is an 83 year old lady diagnosed with multi infarct dementia in 2008. Lives with husband and
the couple are supported by family members.
Husband reports that his wife is getting worse and is frequently waking up at night and getting up.
She misplaces items and then accuses family member of taking them.
Over the last few months she has started to see young people in the trees in the garden. She is
worried in case they fall out.
More recently she is hearing the sound of a child crying which she finds quite distressing and has
also seen the child down the road. Her relatives were aggravating her by insisting to her that her
experiences were unreal.
Current Medications: simvastatin 40mg od, aspirin 75 mg od medical history; nil of note
Mrs DM answers
Medical review to rule out acute on chronic confusion: bloods and MSU
Support husband with advice not to challenge her about these hallucinations as they are real to her.
Undertake carers assessment for husband to identify ways by which he could be supported in her
care.
Medical review
Bloods and MSU - normal
Prescribed Risperdal 0.25mg nocte
licensed for agitation in dementia.
Only prescribed after discussing risks of taking this medication with her relatives and explaining
possible side effects.
Ongoing medical review
Contact was made within first two weeks to get feedback on her response to this medication.
Continued use will be reviewed within 12 weeks.
Social support
Relatives were provided with psycho-education on dementia to inform them on what to expect and
provide them with coping strategies such as gentle reality orientation. 2
Case 2:
Mr MB Scenario
This is an 81 year old man with severe AD dementia. He has been living alone for the last 6 years
since his wife died. He has a carer who attends to his welfare between the hours of 8:30am and 5:30
pm 6 days a week.
He is generally amiable with his main carer but has been known to throw water over her on occasions
out of the blue.
On the one day that this carer is not around a care agency sends in a carer to observe the same routine
as his main carer. He has been known to be verbally and physically aggressive to the “one day a
week carers” and has been known to walk around in the neighbourhood on the days when they attend
to his care.
In spite of his severe expressive dysphasia, severe decline in his power of recall and total dependence
on his carers for his basic hygiene and dressing, he remains able to find his way around in the
neighbourhood.
Mr MB Answers
He was initially placed on Olanzapine by his GP to help address his behaviour to his “one day
carers” but he became excessively sedated on this medication.
This medication was withdrawn and changed to Sodium Valproate which seemed to calm him down
and make him more amenable to care until he pulled a knife on one on the “one day carers”. The
carers withdrew their services.
Next steps
He was admitted to hospital in the interest of his health and safety and safety of others. Recall patient
has no mental capacity to cater for himself
Revise his care package and medical treatment .
Use his hospital admission as an opportunity
to put together a package to accommodate his needs
24 hour supported accommodation for supervision and support he requires.
assess his response to medications aimed at reducing his verbal and physical aggression.
Communicate with primary care clinicians about rationale for any changes and follow up.
Review his physical health
Rule out “acute on chronic” confusional state 3
Case 3:
Mrs A: Scenario
This is an elderly frail female patient with dementia who was known to local community mental
health trust and a private GP. She lived alone, was a life long smoker and was admitted to hospital
with fractured neck of femur. She was extremely anxious on the ward and refused to engage with
rehabilitation .
• Drugs on admission:
– quetiapine, mirtazapine, thyroxine, and pregabalin

Mrs A: Answers
What might have happened to her?
– Delirium identified, anxiety can be reduced by ensuring good sensory awareness by ensuring
hearing aid and her spectacles were available to her .
– Encouraging engagement in activities ( e.g. reminiscence/familiar object around her
– Encouraging good sleep pattern (using milky drinks at bedtime, exercise during the day if possible
) rather than using night sedation
– Advice was offered to family and ward about delirium and management

What else can be explored?


– Questionable prescriptions
– Meds rec revealed that pregabalin had been stopped several months previously.
– Nicotine withdrawal
– Patches were provided
– Adverse drug effects that could be contributing to her conditions
– She had hyponatraemia aggravated by antidepressants,mental state improved when corrected with
delirium resolving.
– Community team was recommended to review quetiapine and mirtazapine after discharge .

Outcome
Anxiety resolved and she was discharged home with care package 4
Case 4:
Mr B Scenario
This 71 year old man, with known history of vascular dementia after a stroke 2 years age=o, was
admitted to hospital with a UTI.
Despite treatment his mood was described by staff as irritable and he began hitting staff and swearing
at them when they got him up in the morning. He settled as the day progressed.
The nurse asks for medication to resolve this issue
Medication review reveals that he was commenced on codydramol for pain 2 weeks prior to the
referral.
Mr B Answers
Investigate, Educate
Check for Underlying infection, Unresolved pain Depression, Adverse effects of prescribed
medication.
Advice staff to check no underlying infection
Gently educate staff on how prescribing drugs without getting to the bottom of the problem is not
helpful .Reminding her how poor prescribing is viewed by CQC and how inappropriate prescribing
can lead to complaints can be helpful
Reflect
• Review pattern to his aggression
– “ABC” (antecedents, behaviour, consequences) chart showed that it only happened with female
staff and was worse the earlier he got up.

Note: his life history information indicated a very private man who had never had any girl friends
and who was a keen supporter of Arsenal
He was also a person who never liked to get up early
Act
• On investigation, he was discovered to be very constipated: laxative were prescribed, fluids and
exercise recommended.
• He was not felt to be in pain and he was put on regular paracetamol instead of co dydramol.
• Staff were given information about his life history which allowed them to modify his care.
– Staff got him up last
– Whenever possible used male staff to treat him
– When helping him wash and dress they used to talk about the latest Arsenal match

Outcome: he became less agg

You might also like